500 onco questions.docx

May 31, 2016 | Author: Melchor Felipe Salvosa | Category: Topics
Share Embed Donate


Short Description

oncology nursing questions...

Description

500 Oncolgy Nursing Questionnaires’ 1. Mr. Smith has laryngeal carcinoma with regional recurrence. During a routine physical exam you notice he

a. 0.5-10%

is experiencing stridor. This is most likely an indication of

b. 10-30%

which of the following?

c. 30-50%

a. Extrathoracic airway obstruction

d. 50-

b. Intrathoracic airway obstruction c. Tumor d. a and c 2. Which of the following is the most accurate measure of dyspnea in the individual with cancer? a. Respiratory rate b. Oxygen saturation c. Arterial Blood Gas level d. Patient self-report 3. A patient who just been diagnosed with lung cancer denies the diagnosis and refuses to hear about it. The patient’s denial behaviour is a. Adaptive b. Maladaptive c. Value neutral d. None of the above Which 4. Support of loss and grief in the person suffering from cancer is primarily concerned with which of the following? a. How to manage symptoms of cancer b. How to increase “masked grief” c. How to identify and aware of losses d. How to use denial to cope with impending loss 5. Which of the following symptoms is least diagnostic of anxiety in the person with cancer? a. Anxiety b. Distractibility c. Worry d. Restlessness 6. When standardized psychiatric interviews and research diagnostic criteria are used, the prevalence of anxiety among cancer patients is approximately which of

7. Early-stage prostate cancer commonly involves surgery or radiation therapy. Diagnostic transurethral resection of the prostate (TURP) commonly causes retrograde ejaculation and may be associated with erectile dysfunction what percentage of the time? a. 10% b. 25% c. 35% d. 50% 8. Chemotherapy agents damage the hair most when it is in which phase of hair growth? a. Anagen b. Catagen c. Telogen d. Transitional 9. Which of the following chemotheraphy agents is least likely to cause hair loss? a. Cyclophosphamide b. Docetaxel c. Vinorelbine d. Etoposide 10. Which of the following is leading cause of cancer death in women in developing counties such as Thailand, Vietnam and Columbia? a. Ovarian Cancer b. Cervical Cancer c. Breast Cancer d. Gastrointestinal Cancer 11. In the Asian culture illnesses such as cancer are believed to be due to which of the following?

the following?

Fourth Year- BSN

Page 1

500 Oncolgy Nursing Questionnaires’ a. An imbalance between yin and yang

a. The total self-appraisal of cancer patients, both men

b. A curse by a spirit/spiritual imbalance

and women

c. An obstruction of chi (an essential life energy)

b. The effects of various treatments on relationships of

d. All of the above

cancer patients with significant others c. Women with gynaecologic or breast cancer or males

12. Effective cancer control is influenced most by which of

with testicular or prostate cancer

the following?

d. The interaction of variables such as age, depression, and activity status on the psychosocial aspects of social

a. Government policy

health

b. Routine chest x-rays c. Hygiene

17. Which of the following systemic treatments for breast

d. A low-fat di

cancer is least likely to cause impairment in sexual function?

13. Nonsteroidal anti-inflammatory drugs have been purported to prevent which of the following?

a. Monthly goserelin injection b. Monthly goserelin injection plus tamoxifen

a. Bladder Cancer

c. Chemotherapy alone

b. Breast Cancer

d. Tamoxifen alone

c. Colorectal Cancer d. Prostate Cancer 18. Two basic nursing interventions for alterations in 14. Which of the following is one of the key issues for

sexual health encountered by cancer patients are:

adult children of cancer patients? a. Disruption of current family relationships

a. Education and counseling

b. Behavior problems

b. Screening and role playing

c. Lack of involvement in decision making regarding the

c. Affective therapy and role modelling

parent’s illness

d. Enhancing reality surveillance and reinforcing

d. Assuming the protector role and shielding parents

personal power

from discussing feelings 19. Sexuality in the cancer patient may be affected by the 15. Women treated for gynaecologic malignancy face a

following factors exept

50% or greater chance of sexual dysfunction due primarily to which of the following.

a. Psychosexual changes associated with mutagenicity b. Physiologic problems of fertility and sterility

a. Changes in body image

c. Psychologic issues such as loss of self esteem and fears

b. Changes in sexual functioning

of abandonment

c. Fertility issues

d. Changes in body appearance resulting from therapy

d. All of the above 20. When documenting patterns of hair loss due to 16. Most empirical studies dealing with the relationship of

chemotherapy, it is important to include all but which of

cancer to self-care concept have focused on

the following assessment criteria a. Family history of male pattern baldness b. Lenght, texture, and curl/wave of hair

Fourth Year- BSN

Page 2

500 Oncolgy Nursing Questionnaires’ c. Condition of the scalp d. Description of patterns of hair loss over entire body

a. Medical and family history b. Type of treatment and side effects experienced c. cancer site and stage of disease

21. For patients who undergo surgery for gastrointestinal

d. Severity or duration of disease

cancer, possible organic sexual dysfunction is most closely associated with which of the following? a. Placement of a colostomy b. Removal of rectal tissue c. Changes in body image d. Responses by family and friends 22. The most frequently addressed factors contributing to cancer-related suicide or euthanasia are a. Pain and other symptom distress b. Advance illness and poor prognosis c. Family history of suicide or personal suicide history d. Hopelessness and loss of self esteem or control 23. Which of the following is least likely to be a cause of anemia in the cancer patient? a. Decreased red cell production b. Iron deficiency c. The primary disease process d. Radiation therapy

24. A program that regards rehabilitation in cancer care as a dynamic rather than a passive process is most likely to emphasize both ongoing reassessment and a. Customary convalescence b. A hospital or community base c. Redefinition of goals d. Frequent nursing referrals 25. Which of the following factors have been found to be most closely related to rehabilitation needs of the cancer patient?

Fourth Year- BSN

26. A 68-year-old woman is diagnosed with thrombocytopenia due to acute lymphocytic leukemia. She is admitted to the hospital for treatment. The nurse should assign the patient (A) to a private room so she will not infect other patients and health care workers. (B) to a private room so she will not be infected by other patients and health care workers. (C) to a semiprivate room so she will have stimulation during her hospitalization. (D) to a semiprivate room so she will have the opportunity to express her feelings about her illness. 27. The nurse teaches a group of mothers of toddlers how to prevent accidental poisoning. Which of the following suggestions should the nurse give regarding medications? (A) Lock all medications in a cabinet. (B) Child proof all the caps to medication bottles. (C) Store medications on the highest shelf in a cupboard. (D) Place medications in different containers. 28. While inserting a nasogastric tube, the nurse should use which of the following protective measures? (A) Gloves, gown, goggles, and surgical cap. (B) Sterile gloves, mask, plastic bags, and gown. (C) Gloves, gown, mask, and goggles. (D) Double gloves, goggles, mask, and surgical cap. 29. A 6-year-old boy is returned to his room following a tonsillectomy. He remains sleepy from the anesthesia but is easily awakened. The nurse should place the child in which of the following positions? (A) Sims’. (B) Side-lying. (C) Supine. (D) Prone. 30. A nursing team consists of an RN, an LPN/LVN, and a nursing assistant. The nurse should assign which of the following patients to the LPN/LVN? (A) A 72-year-old patient with diabetes who requires a dressing change for a stasis ulcer. (B) A 42-year-old patient with cancer of the bone complaining of pain. (C) A 55-year-old patient with terminal cancer being transferred to hospice home care.

Page 3

500 Oncolgy Nursing Questionnaires’ (D) A 23-year-old patient with a fracture of the right leg who asks to use the urinal.

c) assess the client's mental status d) elevate the client's head of the bed

31. Which of these findings in the breast of a patient who is suspected of having breast cancer would support the diagnosis? a) complaints of dull, achy, pain b) palpation of a mobile mass c) presence of an inverted nipple d) area of discoloration skin

37. The nurse on the oncology unit is planning care for the client with colon cancer who is refusing a diagnostic test. Which action is most appropriate for the nurse to take first? a) call the radiology department to let them know the client will not be going to take the test b) speak with the client to determine the reason for refusing the test c) inform the health care provider that the client is refusing the test d) ask the client's spouse why the client is refusing the test

32. A nurse is caring for a client with an internal radiation implant. Which of the followinginstructions is appropriate? a) allow the client to go to the bathroom b) avoid creams and lotions c) visitors are allowed to stay in the room d) the client should remain in bed during the entire duration of treatment 33. How often should a female who is above 40 years old, go for cancer detectionexamination? a) daily b) weekly c) monthly d) yearly 34.The client is receiving internal radiation therapy. The nurse should a) remember to give the badge to the next-shift nurse b) maintain a 30-minute close contact with the patient in a shift c) wear gloves, mask and gown when entering the client's room d) instruct relatives no to visit the client during the entire duration of the treatment 35 A nurse is assessing a client with metastatic breast cancer who reports nocturia, weakness, nausea and vomiting. The client's serum electrolytes include potassium 4.2 mEq/L, sodium 135 mEq/L, calcium 7.0 mEq/L, and magnesium 2.0 mEq/L. Based on the assessment findings,the priority action for the nurse is to: a) start client on fluid restriction b) administer calcium gluconate c) increase the client's IV fluids d) administer Allopurinol 36.. The nurse on the oncology unit enters the room of the client with lung cancer. Which action is most appropriate for the nurse to do first? a) check the client's IV infusion pump and IV fluid rate b) take the client's blood pressure and pulse

Fourth Year- BSN

38. A nurse is admitting a 63-year old male reporting hemoptysis and weight loss. The nurse identifies that the highest priority risk factor for lung cancer for this client is: a) family history of lung cancer b) the client works in a chemical factory c) the client lives in a coal mining area d) the client uses chewing tobacco 39. The nurse is caring for a client with a diagnosis of cancer who is immunosuppressed. The nurse would consider implementing neutropenic precautions if the client's white blood cell count was which of the following? a) 2,000 cells/mm3 b) 5,800 cells/mm3 c) 8,400 cells/mm3 d) 11,500 cells/mm3 40.A nurse is caring for a child after removal of a brain tumor. The nurse assesses the child for which of the following signs that would indicate that brainstem involvement occurred during the surgical procedure? a) inability to swallow b) elevated temperature c) altered hearing ability d) orthostatic hypotension 41. The health education nurse provides instructions to a group of clients regarding measures that will assist in preventing skin cancer. Which statement by a client indicates a need for further instructions? a) I will avoid sun exposure after 3 pm b) I will use sunscreen when participating in outdoor activities c) I will wear a hat, opaque clothing, and sunglasses when in the sun d) I will examine my body monthly for any lesions that may be suspicious

Page 4

500 Oncolgy Nursing Questionnaires’ 42.The client is undergoing radiation therapy to treat lung cancer. Following treatment, the nurse notes erythema on the client's chest and neck, and the client is complaining of pain at the radiation site. The nurse interprets this assessment data a(n): a) allergic reaction to the radiation b) superficial injury to tissue from the radiation c) cutaneous reaction to products formed by the lysis of the neoplastic cells d) ischemic injury, much like pressure ulcer formation. caused by pressure from the linearaccelerator 43. The community nurse is conducting a health promotion program at a local school and is discussing the risk factors associated with cervical cancer. Which of the following, if identified by the client as a risk factor to cervical cancer, indicates a need for further teaching? a) smoking b) multiple sex partners c) first intercourse after age 20 d) annual gynecological examinations 44. The client is diagnosed as having a bowel tumor and several diagnostic tests are prescribed. The nurse understands that which test will confirm the diagnosis of malignancy? a) biopsy of tumor b) abdominal ultrasound c) magnetic resonance imaging d) computed tomography scan 20 45. The nurse is reviewing the laboratory results of a client diagnosed with multiple myeloma. Which of the following would the nurse expect to note specifically in this disorder? a) increased calcium level b) increased white blood cells c) decreased blood urea nitrogen level d) decreased number of plasma cells in the bone marrow 46. The nurse is instructing the client to perform a testicular self-examination. The nurse tells the client: a) to examine the testicles while lying down b.)that the best time for the examination is after a shower c) to gently feel the testicles with one finger to feel for a growth d) that testicular self-examinations should be done at least every 6 months 47. The client with cancer is receiving chemotherapy and develops thrombocytopenia. The nurse identifies which intervention as the highest priority in the nursing plan of care? a) monitoring temperature

Fourth Year- BSN

b) ambulation three times daily c) monitoring the platelet count d) monitoring for pathological fractures 48. The nurse is monitoring the laboratory results of a client preparing to receive chemotherapy. The nurse determines that the white blood cell count is normal if which of the following results were present? a) 2000 to 5000 cells/mm3 b) 3000 to 8000 cells/mm3 c) 5000 to 10000 cells/mm3 d) 7000 to 15000 cells/mm3 49. The community health nurse is instructing a group of female clients about breast self-examination. The nurse instructs the clients to perform the examination: a) at the onset of menstruation b) every month during ovulation c) weekly at the same time of day d) 1 week after menstruation begins

50. The nurse is caring for a client who has undergone vaginal hysterectomy. The nurse avoids which of the following in the care of this client? a) elevating the knee on the bed b) assisting with range-of-motion leg exercises c) removal of antiembolism stockings twice a day d) checking placement of pneumatic compression boots 51. The nurses assesses that the client with cancer is not ready for teaching when the client asks: 1. “Am I going to loose my hair?” 2. “Should I get a second opinion?” 3. “Will this make me really sick?” 4. “Will I have to stop exercising at the gym?” 52. Knowing that chemotherapy affects the taste buds, the nurse would have the client 1. Increase the amount of spices in the food. 2. Avoid red meats. 3. Medicate with Compazine before meals. 4. Eat foods that are hot in temperature. 53. In planning care for a client with a platelet count of 8000 and a WBC of 8000 the nurse can expect to: 1. Remove flowers from the room. 2. Encourage fruits and vegetables. 3. Use strict hand washing technique. 4. Take temperature frequently. 54. The nurse is teaching a client with a WBC of 2000. Which statement the client makes indicates an understanding of the teaching?

Page 5

500 Oncolgy Nursing Questionnaires’ 1. “I will eat fresh fruits and vegetables to avoid constipation.” 2. “I will stay away from my cat.” 3. “I will avoid crowded places.” 4. “I will wash all my fruits and vegetables before I eat them.” 55. In evaluating the client with cancer what best indicates that nutritional status is adequate? 1. Calorie intake 2. Weight is stable 3. Amount of nausea and vomiting 4. Serum protein levels 56. An adult client with newly diagnosed cancer says, “I’m really afraid of dying. Who’s going to take care of my children?” What is the best initial response for the nurse to make? 1. “What makes you think you are going to die?” 2. “How old are your children?” 3. “This must be a difficult time for you.” 4. “Most people with your kind of cancer live a long time.” 57. A client with terminal cancer yells at the nurse and says, “I don’t need your help. I can bathe myself.” Which stage of grief is the client most likely experiencing? 1. Projection 2. Denial 3. Anger 4. Depression 58. The nurse can expect a client with a platelet count of 8000 and WBC count of 8000 to be placed: 1. In a private room. 2. On protective isolation. 3. On bleeding precautions. 4. On neutropenic precautions. 59. Which statement the client makes indicates to the nurse that the client understands external radiation? 1. “I’ll stay away from small children since I am radioactive.” 2. “I won’t wash these marks off until after my therapy.” 3. “I’ll put lotion on my skin to keep it moist.” 4. “I will double flush the toilet each time I use the bathroom.” 60. When teaching and preparing a client for a bone marrow biopsy, the nurse would 1. Check for iodine allergy. 2. Position the client in fetal position with back curved. 3. Have the client sign the consent form. 4. Have the client remain NPO.

Fourth Year- BSN

61. A 28-year-old woman is diagnosed as having iron deficiency anemia. Imferon IM is ordered. The nurse administers it using the Z track technique. The primary reason for administering Imferon via Z track is to: 1. Prolong the action of the drug. 2. Prevent staining of the skin. 3. Improve the absorption rate. 4. Increase the speed of onset of action 62. The nurse is discussing dietary sources of iron with a client who has iron deficiency anemia. The nurse knows the client can select a diet high in iron when she selects which menu? 1. Milkshake, hot dog, beets. 2. Beef steak, spinach, grape juice. 3. Chicken salad, green peas, coffee. 4. Macaroni and cheese, coleslaw, lemonade. 63. A client with iron deficiency anemia is to take ferrous sulfate. She returns to clinic in two weeks. Which assessment by the nurse indicates the client has NOT been taking iron as ordered? 1. The client’s cheeks are flushed. 2. The client reports having more energy. 3. The client complains of nausea. 4. The client’s stools are light brown. 64. A 66-year-old woman is being evaluated for pernicious anemia. What signs and symptoms would the nurse expect to assess in a client with pernicious anemia? 1. Easy bruising. 2. Beefy red tongue. 3. Fine red rash on the extremities. 4. Pruritus. 65. The nurse is caring for a client who is newly diagnosed with pernicious anemia. The client asks why she must receive vitamin shots. What is the best answer for the nurse to give? 1. “Shots work faster than pills.” 2. “Your body can not absorb Vitamin B12 from the stomach.” 3. “Vitamins are necessary to make the blood cells.” 4. “You can get more vitamins in a shot than a pill.” 66. A woman who has had pernicious anemia for several years is seen in the clinic. She tells the nurse that she has a tingling in her arms and legs. What question should the nurse ask initially? 1. “Has your activity level changed lately?” 2. “Has your diet changed recently?” 3. “Have you been sitting more than usual?” 4. “Have you been taking your medicine regularly?”

Page 6

500 Oncolgy Nursing Questionnaires’ 67. A one-year-old is admitted to the hospital with sickle cell anemia in crisis. Upon admission which therapy will assume priority? 1. Fluid administration. 2. Exchange transfusion. 3. Anticoagulant. 4. IM administration of iron and folic acid. 68. The nurse is caring for a 15-month-old child who is newly diagnosed with sickle cell anemia. The mother asks why the child has not had any symptoms before now. The nurse’s response is based on which knowledge? 1. Maternal antibodies have protected the child during the first year of life. 2. Breast milk is a deterrent to sickle cell anemia. 3. The disease does not manifest until the child begins to walk. 4. Elevated fetal hemoglobin levels prevent sickling of red cells. 69. Which statement is essential for the nurse to include in discharge teaching to the parents of a young child who has sickle cell anemia? 1. Do not let her bump into things. She will bruise easily. 2. Notify the physician immediately if she develops a fever. 3. She will need special help with feeding. 4. Observe her frequently for difficulty breathing. 70. The nurse has been teaching the mother of a child with hemophilia about the care he will need. Which statement the mother makes indicates a need for more instruction? 1. “If he needs something for pain or a fever, I will give him acetaminophen instead of aspirin.” 2. “I will take him to the dentist for regular checkups.” 3. “I will keep him in the house most of the time.” 4. “His medical identification bracelet arrived.” 71. A 19-year-old college student reports to the health service with a sore throat, malaise, and fever of four days duration. Examination shows cervical lymphadenopathy and splenomegaly. Temperature is 103oF. Blood is positive for heterophil antibody agglutination test. The nurse knows that infectious mononucleosis is caused by 1. Cytomegalovirus. 2. Beta hemolytic streptococcus. 3. Epstein-Barr virus. 4. Herpes simplex virus I. 72. A client who is diagnosed with infectious mononucleosis asks how he got this disease. The nurse’s response is based on the knowledge that the usual mode of transmission is through 1. Contact with an open wound in the skin. 2. Genital contact.

Fourth Year- BSN

3. Contaminated water. 4. Intimate oral contact. 73. An 8-year-old is admitted to the unit with a diagnosis of acute lymphocytic leukemia. He was receiving a physical exam prior to playing Little League baseball. Numerous ecchymotic areas were noted on his body. His mother reported that he had been more tired than usual lately. The child’s mother says that he has had a cold for the last several weeks. She asks if this is related to his leukemia. The nurse’s response is based on the knowledge that 1. Leukemia causes a decrease in the number of normal white blood cells in the body. 2. A chronic infection such as he has had predisposes a child to the development of leukemia. 3. The virus responsible for colds has been implicated as a possible etiologic agent in leukemia. 4. Having an infection prior to the onset of leukemia is merely a coincidence. 74. A child who is receiving chemotherapy for leukemia has stomatitis. Which of the following nursing care measures is essential? 1. Using dental floss to clean the teeth. 2. Frequent cleaning of the mouth with an astringent mouthwash. 3. Use of an overbed cradle. 4. Swabbing the mouth with moistened cotton swabs. 75. A school age child is receiving chemotherapy for leukemia. Which statement he makes indicates the best understanding and acceptance of what is happening to him? 1. “I hope I won’t loose my hair like the other kids.” 2. “See my new red hat. I like to wear it.” 3. “I want to go see my friend Harold who is in the hospital with meningitis.” 4. “When I’m finished with the chemotherapy, the leukemia will be gone forever.”

76. The nurse is taking the vital signs of the client admitted with cancer of the pancreas. The nurse is aware that the fifth vital sign is: a. Anorexia b. Pain c. Insomia d. Fatigue 77. The nurse is assessing the client admitted for possible oral cancer. The nurse identifies which of the following to be to a late-occuring sx. of oral cancer? a. Warmth

Page 7

500 Oncolgy Nursing Questionnaires’ b. Odor c. Pain d. Ulcer with flat edges 78 The nurse understand that the diagnosis of oral cancer is confirmed with: a. Biopsy b. Gram Stain c. Scrape cytology d. Oral washings for cytology 79. Before administering Methyterxate orally to the client with cancer, the nurse should check the: a. IV site b. Electrolytes c. Blood gases d. Vital signs 80. An elderly is dianosed with ovarian cancer. She has surgery followed by chemotherapywith a fluorouracil (Adrucil) IV. What shoul the nurse do if she notices crystals in the IV medication? a. Discard the solution and order a new bag b. Warm the solution c. Continue the infusion and document the finding d. Discontinue the medication 81. The client with cancer refuses to care for herself. Which action by the nurse would be best? a. Alternate nurses caring for the client so that the staff will not get tired of caring for this client b. Talk to the client and expalin the need for self-care c. Explore the reason for the lack of motivation seen in the client d. Talk to the doctor about the client's lack of motivation 82. A client with cancer develosps xerostomia . The nurse can help alleviate the discomfort the client is experiencing associated with xerostomia by: a. Offering hard candy b. Administering analgesic medications c. Splinting swollen joints d. Providing saliva substitute 83. The nurse notes the pt. care assisstant looking through the personal items of the client with cancer. Which action should be taken by the registered nurse? a. Notify the police department as a robbery b. Report this behavior to the charge nurse c. Monitor the situation and note whether any items are missing d. Ignore the situation until items are reported missing 84. The nurse is caring for a client with a malignancy. The classification of the primary tumor is Tis. The nurse should plan care for a tumor.

Fourth Year- BSN

a. That cannot be assessed b. That is in situ c. With increasing lymph node involvement d. With distant metastasis 85. A client with cancer is to undergo an intravenous pyelogram. The nurse should: a. Force fluids 24 hours before the procedure b. Ask the client to void immediately before the study c. Hold medication that effects the central nervous system for 12 hours pre and post test d. Cover the client's reproductive organs with an x-ray shield. 86. A removal of the left lower lobe of the lung is performed on a client with lung cancer. Which postoperative measure would usually be included in the plan? a. Closed chest drainage b. A tracheostomy c. A mediastenal tube d. Percussion vibration and drainage 87. The nurse is caring for a client with laryngeal cancer. Which finding ascertained in the health history would not be common for this dx? a. Foul breath b. Dysphagia c. Diarrhea d. Chronic Hiccups 88. A client has rectal cancer and is scheduled for an abdominal perineal resection. What should be the priority nursing care during the post-op period? a. Teaching how to irrigate the illeostomy b. Stopping electrolyte loss in the incisional area c. Encouraging a high-fiber diet d. Facilitating perineal wound drainage 89. A client is admitted with a Ewing's sarcoma. Which sx. would be expected due to this tumor's location? a. Hemiplegia b. Aphasia c. Nausea d. Bone pain 90. A client with cancer is admitted to the oncology unit. Stat lab values reveal Hgb 12.6, WBC 6500, k+ 1.9, uric acid 7.0, Na+ 136, and platelets 178,000. The nurse evaluates that the client is experiencing which of the following? a. Hypernatremia b. Hypokalemia c. Myelosuppression d. Leukocytosis 91. A client has cancer of the liver. The nurse should be

Page 8

500 Oncolgy Nursing Questionnaires’ most concerned a. Alteration in nutrition b. Alteration in urinary elimination c. Alteration in skin integrity 92. The nurse on oncology is caring for a client with a white blood count of 600. During evening visitation, a visitor brings a potted plant. What action should the nurse take? a. Allow the client to keep the plant b. Place the plant by the window c. Tell the family members to take the plant home d. Water the plant for the client 93 Which cancer is associated with an increased risk of colorectal cancer? a. Low protein, complex carbohydrates b. High protein, simple carbohydrates c. High fat, refined carbohydrates d. Low carbohydrates, complex protein 94. A client has been hospitalized with a dx of laryngeal cancer. Which factor is most significant in the development of laryngeal cancer? a. A family history of laryngeal cancer b. Chronic inhalation of noxious fumes c. Frequent straining of the vocal cords d. A history of alcohol and tobacco use 95. A client is being treated for cancer with linear acceleration radiation. The physician has marked the radiation site with a blue marking pen. The nurse should: a. Remove the unsightly markings with acetone or alcohol b. Cover the radiation site with loose gauze dressing c. Sprinkle baby powder over the rediated area d. Refrain from using soap or lotion on the marked area 96. While caring for a client with cervical cancer, the nurse notes that the radioactive implant is lying in the bed. The nurse should: a. Place the implant in a biohazard and bag return it to the lab b. Give the client a pair of gloves and ask her to reinsert the implant c. Use tongs to pick up the implant and return it to a leadlined container d. Discard the implant in the commode and doubleflush 97. A client with breast cancer is returned to the room following a right total mastectomy. The nurse should: a. Elevate the client's right arm on pillows b. Place the client's right arm in a dependent sling c. Keep the client's right arm on the bed beside her

Fourth Year- BSN

d. Place the client's right arm across her body 98. A client receiving chemotherapy for breast cancer has an order for Zofran (ondansetron) 8mg PO to be given 30 minutes before induction of the chemotherapy. The purpose of the medication is to: a. Prevent anemia b. Promote relaxation c. Prevent nausea d. Increase neutrophil counts 99. A nursing assisstant assigned to care for a client receiving linear accelarator radium therapy for laryngeal cancer states, "I don't want to be assigned to that radioactive pt." The nurse response by the nurse to: a. Tell the nursing assisstant that the client is not radioactive b. Tell the nursing assisstant to wear a radiation badge to detect the amount of radiation that she is receiving c. Instruct her regarding the use of a lead-lined apron d. Ask a co-worker to care for the client 100. A client with acute leukemia is admitted to the oncology unit. Which of the following would be most important for the nurse to inquire? a. "Have you noticed a change in sleeping habits recently?" b. "Have you had a respiratory infection in the last 6 months?" c. "Have you lost weight recently?" d. "Have you noticed changes in your alertness?"

101. A 35 years old client with ovarian cancer is prescribed hydroxyurea (Hydrea), an antimetabolite drug. Antimetabolites are a diverse group of antineoplastic agents that interfere with various metabolic actions of the cell. The mechanism of action of antimetabolites interferes with: a. cell division or mitosis during the M phase of the cell cycle. b. normal cellular processes during the S phase of the cell cycle. c. the chemical structure of deoxyribonucleic acid (DNA) and chemical binding between DNA molecules (cell cycle– nonspecific). d. one or more stages of ribonucleic acid (RNA) synthesis, DNA synthesis, or both (cell cycle–nonspecific).

Page 9

500 Oncolgy Nursing Questionnaires’ 102. The ABCD method offers one way to assess skin

d. Checking regularly for signs and symptoms of

lesions for possible skin cancer. What does the A stand for?

stomatitis

a. Actinic b. Asymmetry

107. What should a male client over age 52 do to help

c. Arcus

ensure early identification of prostate cancer?

d. Assessment

a. Have a digital rectal examination and prostate-specific antigen (PSA) test done yearly.

103. When caring for a male client diagnosed with a

b. Have a transrectal ultrasound every 5 years.

brain tumor of the parietal lobe, the nurse expects to

c. Perform monthly testicular self-examinations,

assess:

especially after age 50.

a. short-term memory impairment.

d. Have a complete blood count (CBC) and blood urea

b. tactile agnosia.

nitrogen (BUN) and creatinine levels checked yearly.

c. seizures. d. contralateral homonymous hemianopia.

108. A male client complains of sporadic epigastric pain, yellow skin, nausea, vomiting, weight loss, and fatigue.

104. A female client is undergoing tests for multiple

Suspecting gallbladder disease, the physician orders a

myeloma. Diagnostic study findings in multiple myeloma

diagnostic workup, which reveals gallbladder cancer.

include:

Which nursing diagnosis may be appropriate for this

a. a decreased serum creatinine level.

client?

b. hypocalcemia.

a. Anticipatory grieving

c. Bence Jones protein in the urine.

b. Impaired swallowing

d. a low serum protein level.

c. Disturbed body image d. Chronic low self-esteem

105.. A 35 years old client has been receiving chemotherapy to treat cancer. Which assessment finding suggests that the client has developed stomatitis

109. A male client is in isolation after receiving an

(inflammation of the mouth)?

internal radioactive implant to treat cancer. Two hours

a. White, cottage cheese–like patches on the tongue

later, the nurse discovers the implant in the bed linens.

b. Yellow tooth discoloration

What should the nurse do first?

c. Red, open sores on the oral mucosa

a. Stand as far away from the implant as possible and call

d. Rust-colored sputum

for help. b. Pick up the implant with long-handled forceps and

106. During chemotherapy, an oncology client has a

place it in a lead-lined container.

nursing diagnosis of impaired oral mucous membrane

c. Leave the room and notify the radiation therapy

related to decreased nutrition and immunosuppression

department immediately.

secondary to the cytotoxic effects of chemotherapy. Which

d. Put the implant back in place, using forceps and a

nursing intervention is most likely to decrease the pain of

shield for self-protection, and call for help.

stomatitis? a. Recommending that the client discontinue

101. Jeovina, with advanced breast cancer is prescribed

chemotherapy

tamoxifen (Nolvadex). When teaching the client about this

b. Providing a solution of hydrogen peroxide and water

drug, the nurse should emphasize the importance of

for use as a mouth rinse

reporting which adverse reaction immediately?

c. Monitoring the client’s platelet and leukocyte counts

a. Vision changes

Fourth Year- BSN

Page 10

500 Oncolgy Nursing Questionnaires’ b. Hearing loss

115. A male client has an abnormal result on a

c. Headache

Papanicolaou test. After admitting, he read his chart while

d. Anorexia

the nurse was out of the room, the client asks what dysplasia means. Which definition should the nurse

111.. A female client with cancer is being evaluated for

provide?

possible metastasis. Which of the following is one of the

a. Presence of completely undifferentiated tumor cells

most common metastasis sites for cancer cells?

that don’t resemble cells of the tissues of their origin

a. Liver

b. Increase in the number of normal cells in a normal

b. Colon

arrangement in a tissue or an organ

c. Reproductive tract

c. Replacement of one type of fully differentiated cell by

d. White blood cells (WBCs)

another in tissues where the second type normally isn’t found

112. A 34-year-old female client is requesting

d. Alteration in the size, shape, and organization of

information about mammograms and breast cancer. She

differentiated cells

isn’t considered at high risk for breast cancer. What should the nurse tell this client?

116. For a female client with newly diagnosed cancer, the

a. She should have had a baseline mammogram before

nurse formulates a nursing diagnosis of Anxiety related to

age 30.

the threat of death secondary to cancer diagnosis. Which

b. She should eat a low-fat diet to further decrease her

expected outcome would be appropriate for this client?

risk of breast cancer.

a. “Client verbalizes feelings of anxiety.”

c. She should perform breast self-examination during the

b. “Client doesn’t guess at prognosis.”

first 5 days of each menstrual cycle.

c. “Client uses any effective method to reduce tension.”

d. When she begins having yearly mammograms, breast

d. “Client stops seeking information.”

self-examinations will no longer be necessary. 113. Nurse Brian is developing a plan of care for marrow

117. A male client with a cerebellar brain tumor is

suppression, the major dose-limiting adverse reaction to

admitted to an acute care facility. The nurse formulates a

floxuridine (FUDR). How long after drug administration

nursing diagnosis of Risk for injury. Which “related-to”

does bone marrow suppression become noticeable?

phrase should the nurse add to complete the nursing

a. 24 hours

diagnosis statement?

b. 2 to 4 days

a. Related to visual field deficits

c. 7 to 14 days

b. Related to difficulty swallowing

d. 21 to 28 days

c. Related to impaired balance d. Related to psychomotor seizures

114. The nurse is preparing for a female client for magnetic resonance imaging (MRI) to confirm or rule out a

118. A female client with cancer is scheduled for

spinal cord lesion. During the MRI scan, which of the

radiation therapy. The nurse knows that radiation at any

following would pose a threat to the client?

treatment site may cause a certain adverse effect.

a. The client lies still.

Therefore, the nurse should prepare the client to expect:

b. The client asks questions.

a. hair loss.

c. The client hears thumping sounds.

b. stomatitis.

d. The client wears a watch and wedding band.

c. fatigue. d. vomiting.

Fourth Year- BSN

Page 11

500 Oncolgy Nursing Questionnaires’ 119. Nurse April is teaching a client who suspects that

c. Pregnancy complicated with eclampsia at age 27

she has a lump in her breast. The nurse instructs the client

d. Human papillomavirus infection at age 32

that a diagnosis of breast cancer is confirmed by: a. breast self-examination.

124.. A female client is receiving methotrexate (Mexate),

b. mammography.

12 g/m2 I.V., to treat osteogenic carcinoma. During

c. fine needle aspiration.

methotrexate therapy, the nurse expects the client to

d. chest X-ray.

receive which other drug to protect normal cells? a. probenecid (Benemid)

120.. A male client undergoes a laryngectomy to treat

b. cytarabine (ara-C, cytosine arabinoside [Cytosar-U])

laryngeal cancer. When teaching the client how to care for

c. thioguanine (6-thioguanine, 6-TG)

the neck stoma, the nurse should include which

d. leucovorin (citrovorum factor or folinic acid

instruction?

[Wellcovorin])

a. “Keep the stoma uncovered.” b. “Keep the stoma dry.”

125.. The nurse is interviewing a male client about his

c. “Have a family member perform stoma care initially

past medical history. Which preexisting condition may

until you get used to the procedure.”

lead the nurse to suspect that a client has colorectal

d. “Keep the stoma moist.”

cancer? a. Duodenal ulcers

121. A female client is receiving chemotherapy to treat

b. Hemorrhoids

breast cancer. Which assessment finding indicates a fluid

c. Weight gain

and electrolyte imbalance induced by chemotherapy?

d. Polyps

a. Urine output of 400 ml in 8 hours b. Serum potassium level of 3.6 mEq/L

126. Nurse Amy is speaking to a group of women

c. Blood pressure of 120/64 to 130/72 mm Hg

about early detection of breast cancer. The average age of

d. Dry oral mucous membranes and cracked lips

the women in the group is 47. Following the American Cancer Society guidelines, the nurse should recommend

122. Nurse April is teaching a group of women to

that the women:

perform breast self-examination. The nurse should explain

a. perform breast self-examination annually.

that the purpose of performing the examination is to

b. have a mammogram annually.

discover:

c. have a hormonal receptor assay annually.

a. cancerous lumps.

d. have a physician conduct a clinical examination every

b. areas of thickness or fullness.

2 years.

c. changes from previous self-examinations. d. fibrocystic masses.

127. A male client with a nagging cough makes an appointment to see the physician after reading that this

123.. A client, age 41, visits the gynecologist. After

symptom is one of the seven warning signs of cancer. What

examining her, the physician suspects cervical cancer. The

is another warning sign of cancer?

nurse reviews the client’s history for risk factors for this

a. Persistent nausea

disease. Which history finding is a risk factor for cervical

b. Rash

cancer?

c. Indigestion

a. Onset of sporadic sexual activity at age 17

d. Chronic ache or pain

b. Spontaneous abortion at age 19

Fourth Year- BSN

Page 12

500 Oncolgy Nursing Questionnaires’ 128. For a female client newly diagnosed with radiation-

c. Breast cancer requires a mastectomy.

induced thrombocytopenia, the nurse should include

d. Men can develop breast cancer.

which intervention in the plan of care? a. Administering aspirin if the temperature exceeds 102°

132. Nurse Meredith is instructing a premenopausal

F (38.8° C)

woman about breast self-examination. The nurse should

b. Inspecting the skin for petechiae once every shift

tell the client to do her self-examination:

c. Providing for frequent rest periods

a. at the end of her menstrual cycle.

d. Placing the client in strict isolation

b. on the same day each month. c. on the 1st day of the menstrual cycle.

129. Nurse Lucia is providing breast cancer education at

d. immediately after her menstrual period.

a community facility. The American Cancer Society recommends that women get mammograms:

133. Nurse Kent is teaching a male client to perform

a. yearly after age 40.

monthly testicular self-examinations. Which of the

b. after the birth of the first child and every 2 years

following points would be appropriate to make?

thereafter.

a. Testicular cancer is a highly curable type of cancer.

c. after the first menstrual period and annually

b. Testicular cancer is very difficult to diagnose.

thereafter.

c. Testicular cancer is the number one cause of cancer

d. every 3 years between ages 20 and 40 and annually

deaths in males.

thereafter.

d. Testicular cancer is more common in older men.

130. Which intervention is appropriate for the nurse

134. Rhea, has malignant lymphoma. As part of her

caring for a male client in severe pain receiving a

chemotherapy, the physician prescribes chlorambucil

continuous I.V. infusion of morphine?

(Leukeran), 10 mg by mouth daily. When caring for the

a. Assisting with a naloxone challenge test before therapy

client, the nurse teaches her about adverse reactions to

begins

chlorambucil, such as alopecia. How soon after the first

b. Discontinuing the drug immediately if signs of

administration of chlorambucil might this reaction occur?

dependence appear

a. Immediately

c. Changing the administration route to P.O. if the client

b. 1 week

can tolerate fluids

c. 2 to 3 weeks

d. Obtaining baseline vital signs before administering the

d. 1 month

first dose 135. A male client is receiving the cell cycle–nonspecific 131. Nina, an oncology nurse educator is speaking to a

alkylating agent thiotepa (Thioplex), 60 mg weekly for 4

women’s group about breast cancer. Questions and

weeks by bladder instillation as part of a

comments from the audience reveal a misunderstanding of

chemotherapeutic regimen to treat bladder cancer. The

some aspects of the disease. Various members of the

client asks the nurse how the drug works. How does

audience have made all of the following statements. Which

thiotepa exert its therapeutic effects?

one is accurate?

a. It interferes with deoxyribonucleic acid (DNA)

a. Mammography is the most reliable method for

replication only.

detecting breast cancer.

b. It interferes with ribonucleic acid (RNA) transcription

b. Breast cancer is the leading killer of women of

only.

childbearing age.

c. It interferes with DNA replication and RNA

Fourth Year- BSN

Page 13

500 Oncolgy Nursing Questionnaires’ transcription.

b. Maintain an NPO status before the procedure

d. It destroys the cell membrane, causing lysis.

c. Wear comfortable clothing and shoes for the procedure

136. The nurse is instructing the 35 year old client to

d. Drink six to eight glasses of water without voiding

perform a testicular self-examination. The nurse tells the

before the test

client: a. To examine the testicles while lying down

141. A male client is diagnosed as having a bowel tumor

b. That the best time for the examination is after a

and several diagnostic tests are prescribed. The nurse

shower

understands that which test will confirm the diagnosis of

c. To gently feel the testicle with one finger to feel for a

malignancy?

growth

a. Biopsy of the tumor

d. That testicular self-examination should be done at

b. Abdominal ultrasound

least every 6 months

c. Magnetic resonance imaging d. Computerized tomography scan

137. A female client with cancer is receiving chemotherapy and develops thrombocytopenia. The nurse

142. A female client diagnosed with multiple myeloma

identifies which intervention as the highest priority in the

and the client asks the nurse about the diagnosis. The

nursing plan of care?

nurse bases the response on which description of this

a. Monitoring temperature

disorder?

b. Ambulation three times daily

a. Altered red blood cell production

c. Monitoring the platelet count

b. Altered production of lymph nodes

d. Monitoring for pathological fractures

c. Malignant exacerbation in the number of leukocytes d. Malignant proliferation of plasma cells within the bone

138. Gian, a community health nurse is instructing a group of female clients about breast self-examination. The

143.

Nurse Bea is reviewing the laboratory results of a

nurse instructs the client to perform the examination:

client diagnosed with multiple myeloma. Which of the

a. At the onset of menstruation

following would the nurse expect to note specifically in

b. Every month during ovulation

this disorder?

c. Weekly at the same time of day

a. Increased calcium

d. 1 week after menstruation begins

b. Increased white blood cells c. Decreased blood urea nitrogen level

139. Nurse Cecilia is caring for a client who has

d. Decreased number of plasma cells in the bone marrow

undergone a vaginal hysterectomy. The nurse avoids which of the following in the care of this client?

144. Vanessa, a community health nurse conducts a

a. Elevating the knee gatch on the bed

health promotion program regarding testicular cancer to

b. Assisting with range-of-motion leg exercises

community members. The nurse determines that further

c. Removal of antiembolism stockings twice daily

information needs to be provided if a community member

d. Checking placement of pneumatic compression boots

states that which of the following is a sign of testicular cancer?

140. Mina, who is suspected of an ovarian tumor is

a. Alopecia

scheduled for a pelvic ultrasound. The nurse provides

b. Back pain

which preprocedure instruction to the client?

c. Painless testicular swelling

a. Eat a light breakfast only

d. Heavy sensation in the scrotum

Fourth Year- BSN

Page 14

500 Oncolgy Nursing Questionnaires’ b. Restrict fluid intake 145. The male client is receiving external radiation to the

c. Teach the client and family about the need for hand

neck for cancer of the larynx. The most likely side effect to

hygiene

be expected is:

d. Insert an indwelling urinary catheter to prevent skin

a. Dyspnea

breakdown

b. Diarrhea c. Sore throat

150. The home health care nurse is caring for a male

d. Constipation

client with cancer and the client is complaining of acute pain. The appropriate nursing assessment of the client’s

146. Nurse Joy is caring for a client with an internal

pain would include which of the following?

radiation implant. When caring for the client, the nurse

a. The client’s pain rating

should observe which of the following principles?

b. Nonverbal cues from the client

a. Limit the time with the client to 1 hour per shift

c. The nurse’s impression of the client’s pain

b. Do not allow pregnant women into the client’s room

d. Pain relief after appropriate nursing intervention

c. Remove the dosimeter badge when entering the client’s room d. Individuals younger than 16 years old may be allowed to go in the room as long as they are 6 feet away from the client 147. A cervical radiation implant is placed in the client for treatment of cervical cancer. The nurse initiates what most appropriate activity order for this client? a. Bed rest b. Out of bed ad lib c. Out of bed in a chair only d. Ambulation to the bathroom only 148. A female client is hospitalized for insertion of an internal cervical radiation implant. While giving care, the nurse finds the radiation implant in the bed. The initial action by the nurse is to: a. Call the physician b. Reinsert the implant into the vagina immediately c. Pick up the implant with gloved hands and flush it down the toilet d. Pick up the implant with long-handled forceps and place it in a lead container. 149. The nurse is caring for a female client experiencing neutropenia as a result of chemotherapy and develops a plan of care for the client. The nurse plans to:

151 . A 25 year old patient is inquiring about the methods or ways to detect cancer earlier. The nurse least likely identify this method by stating: a. Annual chest x-ray. b. Annual Pap smear for sexually active women only. c. Annual digital rectal examination for persons over age 40. d. Yearly physical and blood examination 152. The removal of entire breast, pectoralis major and minor muscles and neck lymph nodes which is followed by skin grafting is a procedure called: a. Simple mastectomy b. Modified radical mastectomy c. Radiation therapy d. Radical mastectomy 153. Chemotherapy is one of the therapeutic modalities for cancer. This treatment is contraindicated to which of the following conditions? a. Recent surgery b. Pregnancy c. Bone marrow depression d. All of the above 154. The nurse is preparing Cyclophosphamide (Cytoxan). Safe handling of the drug should be implemented to protect the nurse from injury. Which of the following action by the nurse should be corrected? a. The nurse should wear mask and gloves. b. Air bubbles should be expelled on wet cotton. c. Label the hanging IV bottle with ANTINEOPLASTIC

a. Restrict all visitors

Fourth Year- BSN

Page 15

500 Oncolgy Nursing Questionnaires’ CHEMOTHERAPY sign. d. Vent vials after mixing.5 155. Neoplasm can be classified as either benign or malignant. The following are characteristics of malignant tumor apart from: a. Metastasis b. Infiltrates surrounding tissues c. Encapsulated d. Poorly differentiated cells 156. On a clinic visit a client who has a relative with cancer, is asking about the warning signs that may relate to cancer. The nurse correctly identifies the warning signs of cancer by responding: a. “If a sore healing took a month or more to heal, cancer should be suspected.” b. “Presence of dry cough is one of the warning signs of cancer.” c. “A lump located only in the breast area may suggest the presence of cancer.” d. “Sudden weight loss of unexplained etiology can be a warning sign of cancer.” 157. In staging and grading neoplasm TNM system is used. TNM stands for: a. Time, neoplasm, mode of growth b. Tumor, node, metastasis c. Tumor, neoplasm, mode of growth d. Time, node, metastasis 158. Breast self examination (BSE) is one of the ways to detect breast cancer earlier. The nurse is conducting a health teaching to female clients in a clinic. During evaluation the clients are asked to state what they learned. Which of the following statement made by a client needs further teaching about BSE? a. “BSE is done after menstruation.” b. “BSE palpation is done by starting at the center going to the periphery in a circular motion.” c. “BSE can be done in either supine or standing position.” d. “BSE should start from age 20.”

a. b. c. d.

Analgesics before meals Saline rinses every 2 hours Aspirin every 4 hours Bland diet

161. Skin reactions are common in radiation therapy. Nursing responsibilities on promoting skin integrity should be promoted apart from: a. Avoiding the use of ointments, powders and lotion to the area b. Using soft cotton fabrics for clothing c. Washing the area with a mild soap and water and patting it dry not rubbing it. d. Avoiding direct sunshine or cold. 162. Nausea and vomiting is an expected side effect of chemotherapeutic drug use. Which of the following drug should be administered to a client on chemotherapy to prevent nausea and vomiting? a. Metochlopramide (Metozol) b. Succimer (Chemet) c. Anastrazole (Arimidex) d. Busulfan (Myleran) 163 Radiation protection is very important to implement when performing nursing procedures. hen the nurse is not performing any nursing procedures what distance should be maintained fromthe client? a. 1 feet b. 2 feet c. 2.5 feet d. 3 feet 164. The following are teaching guidelines regarding radiation therapy except: a. The therapy is painless b. To promote safety, the client is assisted by therapy personnel while the machine is in operation. c. The client may communicate all his concerns or needs or discomforts while the machine is operating. d. Safety precautions are necessary only during the time of actual irradiation.

159. A client had undergone radiation therapy (external). The expected side effects include the following apart from: a. Hair loss b. Ulceration of oral mucous membranes c. Constipation d. Headache

165. Contact of client on radiation therapy should be limited only to how many minutes to promote safety of the therapy personnel? a. 1 minute b. 3 minutes c. 5 minutes d. 10 minutes

160. Nurse Janet is assigned in the oncology section of the hospital. Which of the following orders should the nurse question if a client is on radiation therapy?

166. Nurse Joy is caring for a client with an internal radiation implant. When caring for the 1client, the nurse should observe which of the following principles?

Fourth Year- BSN

Page 16

500 Oncolgy Nursing Questionnaires’ a. Limit the time with the client to 1 hour per shift b. Do not allow pregnant women into the client’s room c. Remove the dosimeter badge when entering the client’s room d. Individuals younger than 16 years old may be allowed to go in the room as long as they are 6 feet away from the client 167. A cervical radiation implant is placed in the client for treatment of cervical cancer. The nurse initiates what most appropriate activity order for this client? a. Bed rest b. Out of bed ad lib c. Out of bed in a chair only d. Ambulation to the bathroom only 168. A female client is hospitalized for insertion of an internal cervical radiation implant. While giving care, the nurse finds the radiation implant in the bed. The initial action by the nurse is to: a. Call the physician b. Reinsert the implant into the vagina immediately c. Pick up the implant with gloved hands and flush it down the toilet d. Pick up the implant with long-handled forceps and place it in a lead container. 169. The nurse is caring for a female client experiencing neutropenia as a result of chemotherapy and develops a plan of care for the client. The nurse plans to: a. Restrict all visitors b. Restrict fluid intake c. Teach the client and family about the need for hand hygiene d. Insert an indwelling urinary catheter to prevent skin breakdown

c. Incision appearance d. Urine specific gravity 172. A male client is admitted to the hospital with a suspected diagnosis of Hodgkin’s disease. Which assessment findings would the nurse expect to note specifically in the client? a. Fatigue b. Weakness c. Weight gain d. Enlarged lymph nodes 173. During the admission assessment of a 35 year old client with advanced ovarian cancer, the nurse recognizes which symptom as typical of the disease? a. Diarrhea b. Hypermenorrhea c. Abdominal bleeding d. Abdominal distention 174. Nurse Kate is reviewing the complications of colonization with a client who has microinvasive cervical cancer. Which complication, if identified by the client, indicates a need for further teaching? a. Infection b. Hemorrhage c. Cervical stenosis d. Ovarian perforation 175. Mr. Miller has been diagnosed with bone cancer. You know this type of cancer is classified as: a. sarcoma. b. lymphoma. c. carcinoma. d. melanoma.

170. The home health care nurse is caring for a male client with cancer and the client is complaining of acute pain. The appropriate nursing assessment of the client’s pain would include which of the following? a. The client’s pain rating b. Nonverbal cues from the client c. The nurse’s impression of the client’s pain d. Pain relief after appropriate nursing intervention

176 Drugs can cause adverse events in a patient. Bone marrow toxicity is one of the most frequent types of drug-induced toxicity. The most serious form of bone marrow toxicity is: A. aplastic anemia. B. thrombocytosis. C. leukocytosis. D. granulocytosis.

171. Nurse Mickey is caring for a client who is postoperative following a pelvic exenteration and the physician changes the client’s diet from NPO status to clear liquids. The nurse makes which priority assessment before administering the diet? a. Bowel sounds b. Ability to ambulate

177. . Which of the following adverse effects is associated with levothyroxine (Synthroid) therapy? A. tachycardia B. bradycardia C. hypotension D. constipation

Fourth Year- BSN

Page 17

500 Oncolgy Nursing Questionnaires’ 178. The chemotherapeutic DNA alkylating agents such as nitrogen mustards are effective because they: A. cross-link DNA strands with covalent bonds between alkyl groups on the drug and guanine bases on DNA. B. have few, if any, side effects. C. are used to treat multiple types of cancer. D. are cell cycle-specific agents. 179. Hormonal agents are used to treat some cancers. An example would be: A. thyroxine to treat thyroid cancer. B. ACTH to treat adrenal carcinoma. C. estrogen antagonists to treat breast cancer. D. glucagon to treat pancreatic carcinoma. 180. Chemotherapeutic agents often produce a certain degree of myelosuppression including leukopenia. Leukopenia does not present immediately but is delayed several days to weeks because: A. the patient's hemoglobin and hematocrit are normal. B. red blood cells are affected first. C. folic acid levels are normal. D. the current white cell count is not affected by chemotherapy. 181. Currently, there is no way to prevent myelosuppression. However, there are medications available to elicit a more rapid bone marrow recovery. An example is: A. Epoetin alfa (Epogen, Procrit). B. Glucagon. C. Fenofibrate (Tricor). D. Lamotrigine (Lamictal). 182. . Estrogen antagonists are used to treat estrogen hormone-dependent cancer, such as breast carcinoma. Androgen antagonists block testosterone stimulation of androgen-dependent cancers. An example of an androgendependent cancer would be: A. prostate cancer. B. thyroid cancer. C. renal carcinoma. D. neuroblastoma.

Fourth Year- BSN

183. . Serotonin release stimulates vomiting following chemotherapy. Therefore, serotonin antagonists are effective in preventing and treating nausea and vomiting related to chemotherapy. An example of an effective serotonin antagonist antiemetic is: A. ondansetron (Zofran). B. fluoxetine (Prozac). C. paroxetine (Paxil). D. sertraline (Zoloft). 184.. Methotrexate, the most widely used antimetabolite in cancer chemotherapy does not penetrate the central nervous system (CNS). To treat CNS disease this drug must be administered: A. intravenously. B. subcutaneously. C. intrathecally. D. by inhalation. 185. Methotrexate is a folate antagonist. It inhibits enzymes required for DNA base synthesis. To prevent harm to normal cells, a fully activated form of folic acid known as leucovorin (folinic acid; citrovorum factor) can be administered. Administration of leucovorin is known as: A. induction therapy. B. consolidation therapy. C. pulse therapy. D. rescue therapy. 186. Patients undergoing chemotherapy may also be given the drug allopurinol (Zyloprim, Aloprim). Allopurinol inhibits the synthesis of uric acid. Concomitant administration of allopurinol prevents: A. myelosuppression. B. gout and hyperuricemia. C. pancytopenia. D. cancer cell growth and replication. 187. Superficial bladder cancer can be treated by direct instillation of the antineoplastic antibiotic agent mitomycin (Mutamycin). This process is termed: A. intraventricular administration. B. intravesical administration. C. intravascular administration. D. intrathecal administration.

Page 18

500 Oncolgy Nursing Questionnaires’ 188. The most common dose-limiting toxicity of chemotherapy is: A. nausea and vomiting. B. bloody stools. C. myelosuppression. D. inability to ingest food orally due to stomatitis and mucositis. 189. Chemotherapy induces vomiting by: A. stimulating neuroreceptors in the medulla. B. inhibiting the release of catecholamines. C. autonomic instability. D. irritating the gastric mucosa. 190. Myeloablation using chemotherapeutic agents is useful in cancer treatment because: A. it destroys the myelocytes (muscle cells). B. it reduces the size of the cancer tumor. C. after surgery, it reduces the amount of chemotherapy needed. D. it destroys the bone marrow prior to transplant. 191 . Anticipatory nausea and vomiting associated with chemotherapy occurs: A. within the first 24 hours after chemotherapy. B. 1-5 days after chemotherapy. C. before chemotherapy administration. D. while chemotherapy is being administered.

192. Levothyroxine (Synthroid) is the drug of choice for thyroid replacement therapy in patients with hypothyroidism because: A. it is chemically stable, nonallergenic, and can be administered orally once a day. B. it is available in a single 25mg tablet making dosing simple. C. it is not a prodrug. D. it has a short half-life. 193. Your patient has been on a narcotic analgesic for chronic pain from cancer. The dose she has been receiving is no longer bringing about the same pain relief as it once did. The patient asks you why the medicine doesn't work anymore. You explain to her about: A. tolerance. B. potency. C. receptor agonists.

Fourth Year- BSN

D. efficacy. 194. A patient has been receiving chemotherapy to shrink a thoracic tumor. Chemotherapeutic side effects have made eating difficult. The date for surgical removal of his tumor is approaching. Why would the physician initiate total parenteral nutrition (TPN) at this point? A. He wouldn't. TPN will cause the tumor to grow. B. To treat malnutrition and optimize the surgical outcome. C. Because the patient is depressed about his cancer and the chemotherapy side effects. D. It is the standard of all surgeons to initiate TPN prior to surgery. 195. A high-fiber diet is thought to reduce the risk of colon cancer because it: A. absorbs water from the intestinal wall. B. promotes the excretion of bile. C. stops diarrhea. D. is low in kilocalories. 196.Pain tolerance in an elderly patient with cancer would: A. stay the same. B. be lowered. C. be increased. D. no effect on pain tolerance. 197. In administering NSAID adjunctive therapy to an elderly client with cancer, the nurse must monitor: A. BUN and creatinine. B. creatinine and calcium. C. Hgb and Hct. D. BUN and CFT. 198. When administering a narcotic with a non-narcotic to relieve severe cancer pain, the nurse must remember: A. this combination enhances pain relief. B. this combination treats pain both centrally and peripherally. C. the narcotic potentiates action of the nonnarcotic. D. each drug works in its own right. 199. A cancer patient has had a resurgence of severe acute pain. Which of the following routes of medication is most appropriate for this patient? A. oral administration NSAIDS B. oral administration of narcotics

Page 19

500 Oncolgy Nursing Questionnaires’ C. rectal administration of NSAIDS D. injectable pain reliever

replies that it’s breast cancer. Which type of cancer causes the most deaths in women?

. 200Which of the following statements about the use of antidepressants with pain relief in cancer pain is true? A. Antidepressants have no effect. B. Antidepressants enhance the effect of analgesics. C. Antidepressants decrease the effect of analgesics. D. Antidepressants promote more rapid excretion of the medications. 201. The nurse is caring for a recently married, 29-yearold female client, who was diagnosed with acute lymphocytic leukemia (ALL). The client is preparing for an allogencic bone marrow transplant. Which statement by the client demonstrates the understands the informed consent she gave about the diagnosis and treatment? a“I should be able to finally start a family after I’m finished the chemo.” b“I always had a good appetite, even with chemo I shouldn’t have to make any changes to my diet.” c“I’ll have to remain in the hospital for about 3 months after my transplant. d“I’ll only need chemotherapy treatment before receiving my bone marrow transplant.” 202. A client is scheduled to receive methotrexate (Folex). 0.625 mg/kg P.O daily, to treat malignant lymphoma. Before administering the drug, the nurse reviews the client’s medication history. Which drug might interact with methotrexate? a.Digoxin (Lonoxin)

a. b. c. d.

Breast cancer Lung cancer Brain cancer Colon and rectal cancer

204. Client complains of sporadic epigastric pain, yellow skin, nausea, vomiting, weight loss and fatigue. Suspecting gallbladder disease, the physian orders a diagnostic workup, which reveals gallbladder cancer. Which nursing diagnosis may be appropriate for this client? a. b. c. d.

Anticipatory grieving Impaired swallowing Disturbed body image Chronic low self -esteem

205.What is the most important postoperative instruction the nurse must give a client who has just returned from the operating room after receiving a subarachnoid block? a. b. c. d.

“Avoid drinking liquids until the gag reflex returns” “Avoid eating milk products for 24 hours “Notify a nurse if you experience blood in your urine” “Remain supine for the time specified by the physician”

206 A client with colon cancer requires a permanent colostomy because of the tumor. After surgery, the client must learn how to irrigate the colostomy. When irrigating, how far into the stoma should the client insert the lubricated catheter?

b.Theophylline (Slo-phyllin) c.Probenecid (Bennemid) d.

Famotidine (Pepcid)

203.A client with uterine cancer asks the nurse, “Which is the most common type of cancer in women?” The nurse

Fourth Year- BSN

a. b. c. d.

0.25” to 0.5” 1’ to 1.5” 2” to 4” 5” to 7”

207.During the breast examination , which finding more strongly suggests that the client has breast cancer?

Page 20

500 Oncolgy Nursing Questionnaires’ d. a. b. c. d.

Slight asymmetry of the breast A fixed nodular mass with dimpling of the overlying skin Bloody discharge from the nipples Multiple firm, round, freely movable masses that change with the menstrual cycle

208.After cancer chemotherapy, a client experiences nausea and vomiting. The nurse should assign highest priority to which intervention? a. b. c. d.

Serving small portions of bland food Encouraging rhythmic breathing exercise Administering metoclopramide (Reglan) and Dexamethasone(Decadron) as prescribed Withholding fluids for the first 4 to 6 hours after chemotherapy administration

209.A client receiving chemotherapy to treat breast cancer. Which assessment finding indicates a fluid and electrolyte imbalance induced by chemotherapy?

a. b. c. d.

Urine output of 400 ml in 8 hours Serum potassium level of 3.6 mEq/L Blood pressure of 120/64 to 130/72 mm Hg Dry oral mucus membrane and cracked lips

210Which nursing intervention is most appropriate for a client with multiple myeloma? a. Monitoring respiratory status b. Balancing rest and activity c. Restricting fluid intake d. Preventing bone injury 211. An oncology nurse educator is speaking to a women’s group about breast cancer. Questions and comments from the audience reveal a misunderstanding of the aspects of the disease . Various members of the audience have made all of the following statements. Which one is accurate? a. Mammography is the most reliable method for detecting breast cancer b. Breast cancer is the leading killer of women of childbearing age c. Breast cancer requires a mastectomy

Fourth Year- BSN

Men can develop breast cancer

212.Which action, displayed by a grieving husband over his dying wife, would cause the nurse to suggest counselling? a. b.

c. d.

He takes out wedding pictures and memorabilia to show you He refuses to acknowledge his wife’s family and blames them for the client’s current health problem He has already planned his wife’s funeral He is planning to give away his wife’s treasured items to family members

213.The physician orders cystoscopy and random biopsies of the bladder for a client who reports painless hematuria. Test results reveal carcinoma in the situ in several bladder regions. To treat bladder cancer, the client will have a series or intravesical instillations of bacillus CalmetteGuerin (BCG), administered 1 week apart, when teaching a client about BCG, the nurse should mention that this drug commonly causes: a. b. c. d.

Renal calculi Hematuria Delayed ejaculation Impotence

214.A client with suspected lung cancer is suspected lung cancer is scheduled for thoracentesis as a part of the diagnostic workup. The nurse reviews the client’s history for conditions that might contra indicate this procedure. Which condition is contraindication for thoracentesis? a. b. c. d.

A seizure disorder Chronic obstructive pulmonary disease Anemia A bleeding disorder

215.A client is receiving the cell cycle- nonspecific alkylating agent thiootepa(Thioplex), 60 mg weekly for weeks by the bladder instillation as part of a chemotherapeutic regimen to treat bladder cancer. The client asks the nurse how the drug works. How does thiotepa exert its therapeutic effects?

Page 21

500 Oncolgy Nursing Questionnaires’ a. b. c. d.

It interferes with deoxyribonucleic acid (DNA) replication only It interferes with ribonucleic acid (RNA) transcription only It interferes with DNA replication and RNA transcription It destroy the cell membrane, causing lysis

216.A home care nurse assesses for disease complications in a client with bone cancer. The nurse knows that cone cancer may cause which electrolyte disturbances? a. b. c. d.

Hyperkalemia Hyperkalemia Htperkalemia Hyperkalemia

217.After being in remission from Hodgkin’s disease for 18 months, a client develops a fever of unknown origin. The physician orders a blind liver biopsy to rule out advancing Hodgkin’s disease and infection. Twenty four hours after biopsy , the client has a fever, complains of severe abdominal pain and seems increasing confuse. The nurse suspects that this findings result from: a. b. c. d.

Bleeding in the liver caused by the liver biopsy Perforation of the colon caused by the liver biopsy An allergic reaction to the contrast media used during the liver biopsy Normal post-procedural pain with the chnag ewith the level of conscoiysness resulting from the pre existing fever.

218.After the physicians explain the risk and benifits of a clinical trial to a client, the client agrees to participate. Later that day, the client request clarification of the process involved in the clinical trial. As the member of the multi-disciplinary team, how should the nurse respond? a. b. c. d.

Provide the information requested Encourage the client to withdarw from the trial Not provide the information because its bearing in the scope of nursing practice. Tell the client that the information should come from the physician that first presented it to him

Fourth Year- BSN

219.After receiving chemotherapy for lung cancer a clients platelet count falls to 98 000/ul. What term should the nurse used to describe this low platelet count: a. Anemia bLeukopenia c. Thromcytopenia dNeutrophenia 220.To combat the most common adverse effect of chemotherapy , the nurse must administer an: a. b. c. d.

Antiemetic Antimetabolite Antibiotic Anticoagulant

221.A client is hoispitalized with cell carcinoma of the lung. To manage severe pain , the physicians prescribed a continous IV infusion of morphine. Which formula should the nurse use to check that the morphine dose is appropriate for the client? a. b. c. d.

1 mg/ kg of body weight 5 mg/ kg of body weight 5 mg/ 70 kg of body weight 10 mg/ 70 kg of body weight

222.The nurse is providing best cancer education at the community facility. The American cancer society recommends that women get mammograms: aYearly after age 40 b After the birth of the first child and every two years there after. C After the first menstrual period and annually there after. dEvery three years between ages 20 and 40 and annually there after. 223.The nurse is caring for a patient with a long term central venous catheter. Which care principle is correct? aUse clean technique when assessing the port with a needle

Page 22

500 Oncolgy Nursing Questionnaires’ b If the needle to comes contaminated before accessing the port, clean the needle with povidone iodine solution. C Clean the port with an alcohol pad before administering IV fluid through the catheter 224.A client has just begun treatment with busulfan, 4 mg by mouty daily, for chronic myelogenous leukemia. The client receives busulfan until his WBC count falls to between 10 000/mm3 and 25 000/mm3. Then the blood is stopped. When should treatment resume? a. When the WBC falls to 5000 mm3 b. When loss hair begins to grow back c. When the WBC rises to 50 000 mm3 d. When the client displays anemia 225. Which of the following is characteristic of a benign neoplasm? a. Rapid growth b. invasive c. noncohesive d. well-defined borders 226. Tina Blackey was recently diagnosed with breast cancer. She asks the nurse what steps she can take to boost her immune system. The best response by the nurse is: a. “Refuse chemotherapy; it weakens the immune system.” b. “Learn techniques to relieve or manage stress more effectively.” c. “Ask your doctor to prescribe an antidepressants.” d. “Research herbal remedies.” 227 Which of the following explanations should the nurse utilize regarding the magnetic resonance imaging test used to diagnose a suspected brain tumor? a. b.

“This test sound waves.” “Before this test, the physician will inject you with a radiopaque dye and then films will be taken.” c. “The machine for this is like a tube, and you will lie inside. It may make thumping noises.” d. “You must drink a radioactive substance before this test.” 228. Jean Lawrence is undergoing diagnostic testing as part of the staging process. She tells the nurse, “I am so worried they will find cancer everywhere; what will

Fourth Year- BSN

happen if I die before my children are grown up?” The most appropriate response by the nurse is: a. “There is no way of knowing what the future holds.” b. “No matter what happens, your children will be fine.” c. “You sound like you are afraid.” d. “Don’t worry until you know the results.” 229. Which of the following client statements indicates the need for further client-teaching about chemotherapy? a. “Chemotherapy is a local treatment.” b. “I may experience nausea, vomiting, diarrhea, and loss during the treatments.” c. “I may receive both oral and IV medications.” d. “I may receive a combination of drugs over different periods of time.” 230. Which of the following statement is true about cell-kill hypothesis? a. It explains the body’s mechanism of protection from carcinogens. b. It forms the basis for selecting specific classes of chemotherapeutic drugs. c. It accounts for the severity of side effects. d. It is the reasons why several courses of chemotherapy are given. 231. A example of a chemotherapy catheter that is tunneled under the skin on the chest into the subclavian vein is a. Hickman b. peripherally inserted central catheter c. mediport d. port-a-cath 232. Assessing for arrhythmias, gallops, and congestive heart failure is an especially important nursing intervention for a client receiving: a. Vinblastin (velban) b. Doxorubicin (adriamycin) c. 5-fluorouracil (5-FU) d. Vincristine (oncovin) 233.Marlene Morrison is scheduled to start chemotherapy. Which statement by Mrs. Morrison indicates a need for further client-teaching? a. b. c.

“I will have to drink plenty of fluids each day.” “visits from my school-aged grandchildren will help my spirits.” “I will need extra rest during this period of time.”

Page 23

500 Oncolgy Nursing Questionnaires’ d.

“I may need extra help at home to clean and prepare meals.”

234.Shelley Nordant is scheduled for surgery to remove a cancerous tumor. She remarks to the nurse, “Why have surgery? I am going to have chemotherapy just same.” The most appropriate response by the nurse is to: a. b. c. d.

Tell Mrs. Nordant that surgery must be performed Encouraged her to confer with the surgeon and the oncologist about the surgery Tell her that the combination of surgery and radiation may lead to a cure Document her concerns and give her reassurance

235.Which of the following statements best describes brachytherapy? a. b. c. d.

Brachytherapy involves placing radioactive materials at some distance from the tumor Brachytherapy involves placing radioactive materials directly into the tumor site Brachytherapy is a holistic, nontradional cancer treatment Brachythaerapy is also referred to as teletherapy

236.Which of the following nursing actions is most appropriate when caring for a client with unsealed implanted radioisotope? a. b. c. d.

Wear an abdominal apron Have the other client in the room wear a monitoring device to measure exposure Pick up any dislodged implants found in the bed linen with your sterile gloved hand Dispose of client’s body fluids in specially marked containers

237 Which of the following statement indicates that the client-teaching prior to an external radiation was successful? a. b.

“if I have pain at my treatment site, I will apply an ice pack.” “I will not have any intimate sexual contact while I am receiving treatments.”

Fourth Year- BSN

c. d.

“I should protect my skin from sun exposure during treatments.” “I will wash the skin in the treatment area daily using soap and hot water.”

238. George Houng is receiving combination therapy of alpha-interferon with another chemotherapeutic agent. Which statement indicates that the client and family further teaching from the nurse? a. b. c. d.

“I will observe my husband for any changes in his mental status.” “I understand how to give the drug subcutaneously.” “I can have severe flulike symptoms while taking these drugs.” “I will limit my fluid intake.”

239.Stuart Eggleton, a client with prostate cancer, is trembling and is pacing in his room. His blood pressure and pulse are slightly elevated. Mr. Eggleton may be experiencing: a. b. c. d.

Some type of drug reaction Anxiety Body image disturbance Fear of unknown

240. Which of the following statements indicates that the client has misunderstood client-teaching about alopecia from chemotherapy? a. b. c. d.

“I can wear a colorful scarf or turban.” “I have already bought a wig.” “My neighbor will be driving me to the ‘look good, feel better’ program.” “After the chemotherapy is finished, my hair will grow back the same as it was before the treatments.”

241. The partner of a client with cancer has been provided with guidelines regarding when to call for help. Which of the following statements by the client’s partner indicates that further teaching about the guidelines is needed? a. b.

“I’ll contact the physician if I see any new bleeding from her rectum.” “If her oral temperature is 100 F, I’ll notify the physician.”

Page 24

500 Oncolgy Nursing Questionnaires’ c.

“If she becomes withdrawn and cries frequently, I’ll notify the physician.” “I’ll contact the physician if I notice a change in her eating patterns.”

b.

242. Which of the following nursing assessments would indicate an improvement in a cancer client with nursing diagnosis altered nutrition: less than body requirements?

d.

d.

a. b. c. d.

Serum albumin less than 3.0 A PPD response of more than 5 mm of induration A decrease in urinary creatinine level An anthropometric measurement of 75% of the standard

243. Which of the following nursing interventions would be most helpful in managing the nausea and vomiting of a client with cancer? a. b. c. d.

Instruct the client to drink plenty of fluids with each meal Instruct the client to lie down for an hour after eating Encourage the client to eat dry foods Instruct the client to follow a high-fat food plan

244. Which of the following nursing actions is contraindicated for the client with leukemia? a. b. c. d.

Assess the client’s oral mucous membranes daily Instruct the client to brush and floss the teeth three ties per day Prevent dry mouth by using moisturizing agents such as blistex Teach client to use smooth rather than sharp utensils for eating

245. The doctor has told Javier Rodriguez and his family that he has about 6 to 9 months to live. The family asks the nurse about making funeral arrangements ahead of time. The most appropriate response by the nurse is: a.

“Many people do make arrangements ahead, but you might upset Mr. Rodriguez if you tell him about the plans.”

Fourth Year- BSN

c.

“If you would like to do this, you should try to include Mr. Rodriguez in the planning.” “Don’t give up hope, he may still respond to therapy.” “You should wait until Mr. Rodriguez dies before making funeral plans.”

246. Which strategy would not be recommended to a cancer client with problems eating? a. b.

Eat icy- cold foods semisoft and liquid foods Eat frequent, small meals spicy, warm foods

c. eat d. eat

247. Dana Biencarz has lung cancer and is experiencing facial and arm edema. The nurse should suspect that Mrs.Biencarz has: a. b.

Allergic drug reaction c. sepsis Pericardial effusion superior vena cava syndrome

d.

248. Rita Lopez has metastatic breast cancer and is admitted to the hospital for pain management. During the night she becomes incontinent, which has never occurred before. The appropriate action by the nurse is to: a. b. c. d.

Recognize that the incontinence occurred as a result of over sedation Notify the physician that spinal cord compression is suspected Realize that incontinence can occur in metastatic bone disease Wait until the morning to see if the incontinence recur

249..The nurse is caring for a client who has just a modified radical mastectomy with immediate reconstruction. She’s in her 30’s and has two young children. Although she worried abot her future, she seems to be adjusting well to her diagnosis what should the nurse do to support her coping? a.

Tell the client spouse or partner to be supportive while she recover

Page 25

500 Oncolgy Nursing Questionnaires’ b. c. d.

Encourage the client to procede with the next phase of treatment Recommend that the client remain cheerful for the sake of her children Refer the client to the American cancer society reach for recovery program or another support program.

250. Which lab test is it most important to monitor for a client with lung cancer that has ectopic sites? a.

T3

b. T4 d. blood culture

c. serum glucose

SITUATION; Nurse Tin-Tin is entrusted with the task of promoting nursing care in client with various surgical conditions. 251. Physical preparation for a client schedule for cystectomy and ilial conduit includes the administration of? A. B. C. D.

Intravenous vitamins C Neomycin sulfate Bladder analgesic Urinary antibiotic

252. Which of the following methods of assessment is correct to assess distention in a post operative client? A. B. C. D.

Palpate the perineal area Palpate the epigastric area Percuss and palpate the lower abdomen Percuss and palpate the upper abdomen

253. After thyroidectomy, which of the following intervention is the highest priority of the nurse? A. Monitor the client’s vital signs every 4h. B. Observe the dressing at the back of the neck for the presence of blood C. press gently around the incision to assess for subcutaneous emphysema D. positon the client laterally to promote drainage of secretions from the mouth 254. Following cataract extraction, which client activity is allowed?

Fourth Year- BSN

A. coughing deeply to prevent pneumonia B. Bending from the waist to prevent contracture deformity C. Remainins in a lying down supine position for 48h after surgery D. Deep breathing to promote lung expansion 255. What is the highest priority, in terms of goal of care, by the nurse after a client’s prostatectomy? A. B. C. D.

To prevent infection To prevent bleeding To prevent impotence To prevent respiratory distress

SITUATION; A nurse is providing care to clients with acquired immune deficiency syndrome. 256. The treatment of AIDS involves a palliative approach. what does this approach means? A. Curing the disease B. Relieving the client’s discomfort C. Providing rehabilitative support to the client’s family D. Promoting recovery 257. A positive diagnosis for HIV infection is made based on the result of which of the following test? A. B. C. D. E. F. G. H.

ELISA Western blot WBC count CD4 cell count A and C A and B A,B and C A, B and D

258. Which principles of isolation should the nurse implement when obtaining the blood pressure of a client who has AIDS? A. Use of mask B. Wearing clean gloves C. Hand washing D. Use of clean gown 259. Which of the following practices limits a client’s exposure to thehuman immunodeficiency virus?

Page 26

500 Oncolgy Nursing Questionnaires’ A. B. C. D.

Using a condom during sexual intercourse Limiting sexual contact only to the spouse Donating 200ml of blood Having sexual intercourse only to those without HIV antibodies

260. Which of the following symptoms are most commonly seen in clients with AIDS? A. B. C. D.

Low-grade fever and persistent diarrhea Cough and abdominal pain Projectile vomiting and jaundice Headache and rash

SITUATION; The nurse is expected to perform her function in the operating room with outmost care. 261. A client is to undergo an exploratory laparotomy for vague abdominal discomforts that his lead to abdominal rigidity and severe pain. As the surgeon opens the client’s abdomen, which of the following instruments shoul the nurse anticipate the surgeon to use? A. B. C. D.

Kocher Mixtner Kelly Army navy

262. Surgical closure of the skin surfaces usually involes the nurse of which type of needle? A. B. C. D.

Straight needle Curved needle Rounded needle “eye less” needle

263. The surgeon closes the client’s abdominal tissue layer. The nurse knows that closure of the abdominal tissue layers begins with the peritoneum, extra peritoneal fat, transversalis facia,followed by? A. B. C. D.

Deep facia, muscle subcutaneous tissue ,skin Muscle, deep facia, subcutaneous tissue, skin Subcutaneous tissue, deep facia, muscle, skin Skin, muscle, subcutaneous tissue deep facia

264. Following cataract surgery, which aspect of nursing care is the highest priority of the nurse? A. Nutrition B. Airway

Fourth Year- BSN

C. Safety D. Fluids 265. To facilitate drainage of secretions from the operative site, the nurse should? A. Turn the client to the unoperative side every 2-3h B. Turn the client to the operative side every 2-3 h C. Keep the client supine until wound healing D. Place the client on semi-fowler’s position at least once every shift SITUATION; Mr. Katrina Co, R.N., is assigned to take care of clients with traumatic injuries,electrolyte disturbances and adrenal tumor. 266. A client with chest trauma has a chest tube in place. after the removal of the tube, the client’s PaO2 value is 85mmHg. The nurse should interpret this value to indicate that the oxygen level? A. Is normal B. Need to be increased C. Need to be reported to the physician immediately D. Indicates that the client needs to keep breath and cough 267. Which of the following methods is appropriate to documents the amount of drainage from a client’s chest tube? A. Aspirate the contents of the pleur evac and measure the drainage B. Empty the pleur evac during every shift C. Mark the fluid level by placing a tape on the collection chamber of the pleur evac D. Use a new set of pleur evac every shift 268. Pulmonary capillary wedge measure is an indirect measure of left ventricular end diastolic pressure. This provides data on ventricular contractility. which catheter is appropriate to measure PCWP? A. B. C. D.

Swan Ganz CVP Tenckhoff Grosshong

Page 27

500 Oncolgy Nursing Questionnaires’ 269. Which electrolyte imbalance occurs in a client with acute renal failure? A. B. C. D.

Hypocalcemia and hypophosphatemia Hypomagnesemia and hypocalcemia Hypomagnesemia and hypophophostemia Hyponatremia and hyperkalemia

270. Which of the following interventions take priority in a client with pheochromocytoma? A. Counting the respiratory rate for one full minute B. Monitoring the blood pressure C. measuring the urine output in 24h D. Obtaining the axillary temperature every hour SITUATION; Care of client with TURP. 271. To promote bladder decompression and to prevent bladder decompression after transurethral resection of the prostate, which intervention should be the nurse’s priority? A. B. C. D.

Monitoring the client for signs of hemorrhage Maintaining patency of the cystostomy tube Assessing the client for signs of infection Ensuring patency of the continuous bladder irrigation

272. How should the nurse determine the urine outputof a client on continuous bladder irrigation? A. Deduct the amount of the irrigation solution instilled from the total amount of output B. Deduct the amount of IV fluid infusion from the total fluid intake of the client C. Add the amount of all fluid intake then deduct all the amount of output D. Measure amount of output from the bladder irrigation 273. Which of the following is most common complication of transurethral resection of the prostate in the early post operative period. A. B. C. D.

Impotence Infection Bleeding Urinary retention

Fourth Year- BSN

274. To promote hemostasis in a client with transurethral resection of the prostate the client’s retention catheter is secured? A. B. C. D.

To the lower abdomen To the leg To the bed To the intravenous pole

275. Which of the following manifestations should the nurse instruct the client to report to the physician immediately after undergoing TURP A. B. C. D.

Burning on urination Decreased urinary stream Pink- tinged urine on initial voiding Urine output of 30ml/h

SITUATION: Mr. Jeremy Pontanes, R.N., is entrusted with the task of promoting nursing care in client with disturbance in the cardiovascular system. 276. Which of the following characteristic of pain will help differentiate angina pectoris from myocardial infarction in terms of management? A. B. C. D.

Duration Location Severity Cause

277. Which of the following is an early sign of heart failure? A. B. C. D.

Neck vein distention Ankle edema Dsypnea Hepatomegaly

278. In a client with myocardial infarction, activity is gradually increased based on which of the following factors? A. B. C. D.

Amount of weight loss Severity of dsypnea Degree of edema Presence of absence of cyanosis

279. A client with history of myocardial infarction was admitted due to dsypnea. The serum potassium level is

Page 28

500 Oncolgy Nursing Questionnaires’ 2.6mEq/L. Which of the following doctor’s ordere should the nurse question? A. B. C. D.

Propanolol Furosemide Morphine Oxygen 1-21/min

280. The client is placed on a moderate sodium restricted diet, which of the following food is allowed? A. B. C. D.

French fries Ice cream Catsup Apples

SITUATION; Nurse Henry is entrusted with the task of promoting nursing care in client with disturbances in the gastrointestinal tract. 281. Cholangiography allows for the visualization of? A. B. C. D.

Pancreas and large intestine Liver and gallbladder Pancreatic duct and hepatic duct Common bile duct and hepatic duct

282. Which of the following beverages should the nurse instruct a client who has been hospitalized due to peptic ulcer disease to avoid? A. B. C. D.

Mineral water Apple juice Wine Milk

283. The most serious complication of the use of a nasogastric tube is? A. B. C. D.

Fluid and electrolyte imbalance Ulceration Aspiration pneumonia Constipation

284. Which statement made by a client with ulcer indicates an understanding of the need for ulcer medications? A. “Anatacids can be taken with other drugs” B. “Tagament will decrease the acid in my stomach” C. “Maalox will coat my stomach D. “I can take my antacids with aspirin” 285. Which nursing action is of the highest priority in a client with nasogastric tube after gastric surgery?

Fourth Year- BSN

A. Check the tube placement by instilling 10ml of water into the tube B. Change the tube every 24-48h. C. Ensure that the tube is connected to continuous suction D. Assess the client for abdominal cramps and decreased bowel sounds. SITUATION; Nurse Madonna is entrusted with the task of promoting nursing care in client with disturbances in the respiratory system. 286. When a chest tube is being removed, the nurse should instruct the client to? A. B. C. D.

Cough vigorously Deep breath Inhale and hold breath Exhale deeply

287. Which the following equipment does now interfere with eating or talking and can best deliver oxygen in a hypoxic client.? A. B. C. D.

Oxygen tent Oxygen mask Nasal cannula Endotracheal tube

288. Which reading of pulmonary capillary wedge pressure (in mmHg) needs to be reported to the physician immediately? A. B. C. D.

5-10 15-20 25-30 35-40

289. Which of the following client is at most risk for tuberculosis? A. B. C. D.

A 35 –year old newly married A 68-year old client A 12 year-old child A 17 year-old adolescent

290. Which of the following statement best describes the primary purpose of pursed-lip breathing? A. B. C. D.

increase oxygen inhalation Maximize diaphragmatic extension Facilitate inhalation of carbon dioxide Minimize the use of intercostals muscles

SITUATION; Mr. Ronald is entrusted with the task of promoting nursing care in client with disturbances in the biliary system.

Page 29

500 Oncolgy Nursing Questionnaires’ 291. Which assessment finding best indicates deterioration of liver function? A. B. C. D.

Anorexia Constipation Weight gain Lethargy

292. Which of the following aspect of care should have the highest priority for a client admitted due to cholecystitis? A. B. C. D.

Comfort Hydration Health instruction Obtaining thorough history

293. Which of the following laboratory values is NOT expected in a client with pancreatitis? A. B. C. D.

Serum magnesium 3.2mEq/L Serum glucose 45mg/dl Serum potassium 5.5mEq/L Serum calcium 8mg/dl

294. Which client is the best candidate for the use of insulin pump? A. B. C. D.

A 42 year-old businessman A 35 year-old working single mother A 12 year-old mentally retarded An 82 year-old cognitively impaired

295. Which statement by a client with ulcer indicates understanding of the medication therapy? A. B. C. D.

Antacids can be taken with other drugs Tagamet will decrease the acid in my stomach Maalox will coat my stomach I can take my antacid with asprin

SITUATION; Nurse jethro is entrusted with the task of promoting nursing care in client undergoing surgery. 296. To decrease the anxiety of the client and his/her family about an impending surgery, the nurse should? A. Describe the details of the surgery to the client and family B. Provide general information and answer appropriate question of the client and family C. Empower the client to make decision by obtaining the client’s consent D. Give full reassurance that the surgical team is experienced to handle the surgery

Fourth Year- BSN

297. Which of the following procedure for using an incentive spirometer is appropriate for the nurse to provide the client with during pre operative instructions? A. Inhale completely and exhale rapidly through the spirometer B. Exhale completely take slow deep breath through the spirometer, hold breath then exhale through purse lip C. Inhale completely through the spirometer, exhale and hold breath D. Exhale through the spirometer, hold breath then inhale 298. The stage of surgical anesthesia proceeds in which of the following order? A. Excitement, surgical anesthesia,loss of consciousness and danger stage B. Loss of consciousness, excitement, surgical anesthesia and danger stage C. Surgical anesthesia, excitement, loss of consciousness and danger stage D. Loss of consciousness, surgical anesthesia, excitement and danger stage 299. What common side effect should the nurse monitor in a client who just received epidural anesthesia? A. B. C. D.

Fever Bradycardia Hypotension Pallor

300. Which of the following complications can result from the use of general anesthesia during surgery? A. B. C. D.

Fluid loss Peripheral retention Atelectasis Muscle rigidity

SITUATION; Nurse Santini is taking care of Jay, a client who underwent nephrectomy. 301. Which of the following observation is expected in a post nephrectomy client? A. B. C. D.

Urine output is 20ml/h Drak red urine on the third post operative day Urine specific gravity: 1.003 Urinary drainage from the surgical wound a week after surgery

Page 30

500 Oncolgy Nursing Questionnaires’ 302. An appendectomy is usually performed on a client for kidney transplant to provide adequate space for the transplanted kidney, which is usually placed in the clients. A. B. C. D.

Left lower quadrant Right lower quadrant Right upper quadrant Left lower quadrant

303. Hourly monitoring of the client’s urine in essential following nephrectomy to assess a common signs of kidney transplant rejection which is? A. B. C. D.

abdominal pain decreased urine output Decreased blood pressure Anemia

304. Which of the following laboratory data indicates the adverse effect of anti kidney rejection medication administered to a client A. B. C. D.

Platelet count 200,00/mm3 WBC count 3,800/mm3 RBC count 5.1 million/mm3 Hemoglobin 14mg/dL

305. Which of the following laboratory data is the most accurate blood test for determining dehydration? A. B. C. D.

Urine specific gravity Urine osmolarity Creatinine clearance BUN

SITUATION: Care of client with respiratory disorders. 306. Oseltamivir (tamiflu) is most effective if Tken within how many hours after the onset of the symptoms off influenza AH1N1? A. B. C. D.

48 h 72h 96h 100h

307. The priority goal of therapy for a client with chronic obstructive pulmonary disease is to? A. Limit fluid administration B. Provides higher concentration of oxygen

Fourth Year- BSN

C. Improve ventilation D. Maintain acid based balance 308. Which of the following foods should the nurse encourage the client with tuberculosis and osteoporosis to include in the diet? A. B. C. D.

Soft drinks Orange juice Beans Yogurt

309. When communicating with the client with asthma and Wernicke’s aphasia, the nurse should? A. Provide adequate time for the client to respond B. Face the client and speak loudly C. Use short and simple words and sentences D. Provide a pencil and writing pad for the client 310. While performing respiratory assessment among elderly clients, the nurse was able to obtain cues and possible cases of elderly abuse. Which client is most at risk for elderly abuse? A. A 68 year-old cognitively impaired male living alone B. A 72 year-old female living with her spouse and children C. A 65 year-old male living with the household help D. A 67 year-old female living on her own SITUATION; The following question pertain to clients with tubes. 311. A client just underwent choledoccholithotomy and has a T-tube in place. She calls the nurse and complains that the skin around the T-tube is excruciated. The nurse should assess the skin and? A. B. C. D.

Reinforce the dressing Apply antiseptic solution Change the dressing Use a skin barrier around the t-tube site

312. To promote continuous and proper functioning of chest tube attached to a client, the nurse should keep the collection device?

Page 31

500 Oncolgy Nursing Questionnaires’ A. B. C. D.

Above the client’s chest Beside the client Below the level of the client’s chest At floor level

313. After receiving the end shift report that a client’s Ttube has a total drainage of 120 ml in 24h, which of the following intervention is the highest priority of the nurse? A. Refer the situation to the physician immediately B. document the amount on the client’s kardex C. Check the tube for the kinks and proper placement of the drainage bag D. Empty the drainage bag and observe the client further. 314. Following the surgical creation of a permanent colostomy, the client call the nurse and ask why there is no drainage from the colostomy. Which nursing intervention is priority at this time? A. Call the operating room and schedule the client for repeat surgery. B. Notify the physician immediately C. Check the colostomy for obstruction D. Explain to the client that this is expected in the next 2-3 days 315. Which of the following characteristic of pain indicates gallbladder disease? A. Left epigastria pain with cullen’s sing B. Right lower quadrant pain with bloody diarrhea C. Right upper quadrant pain that radiate to the shoulder blade D. Lower sterna pain that resembles indigestion. SITUATION; Mr jose dela cruz, age 68mis admitted to the hospital due to fever, cough and dyspnea. His medical history reveals long standing COPD. 316. The caregiver of a client with COPD ask the nurse “Why can’t we just increase the client’s oxygen to help her breath easily” the appropriate response of the nurse is? A. “let me notify the physician first” B. “Increasing oxygen levels may decrease the client’s urge to breath

Fourth Year- BSN

C.

“The amount of oxygen admitted is computed based on the clients weight” D. “It is better if we try it for a few minutes first” 317. The nurse is providing pre-operative instruction in diaphragmatic breathing to a client with COPD. an indication that the client understand the nurse’s instruction is that he? A. Cough before breathing B. Inhale through the nose and hold breath for 12s before he exhale C. Exhale completely before increasing the rate of breathing D. Inhale through the mouth 318. In a client with COPD, cor pulmonale can possibly result in the enlargement of which of the following organs? A. B. C. D.

Heart and liver Heart and stomach Liver and gallbladder Lungs and stomach

319. To limit the client’s oxygen needs,a client with COPD should be? A. B. C. D.

Served three large meals a day Given six small meal a day Forced to drink large amount of fluid Instructed to immediately sleep after meal

320. The nurse is providing care to a client with a tracheostomy tube in place. The nurse notes that his/her breathing has become noisy. Which intervention in a nursing priority? A. B. C. D.

Adjust the cuff pressure Reposition the client Suction the tracheostomy Notify the physician

SITUATION; Several clients are undergoing diagnostic procedures. 321. A client scheduled for electroencephalogram (EEG) is being served breakfast. On her tray are the following: scrambled egg, fried rice, dried fish and cup of coffee. What should the nurse do? A. Withhold the meal

Page 32

500 Oncolgy Nursing Questionnaires’ B. Remove the fried rice C. Remove the dried fish D. Remove the cup od coffee 322. The use of the snellen’s chart help to evaluate cranial nerve: A. B. C. D.

VI IV III II

323. A client has a Glasgow coma scale score of 4. This is interpreted to mean that the client is? A. B. C. D.

Awake Lethargic Oriented Unresponsive

324. Which of the following diagnostic test does NOT require informed consent? A. B. C. D.

Colonoscopy Electromyelography Electro encephalography Liver biopsy

325. Which client does NOT have any risk from undergoing MRI? A. B. C. D.

A 35 year-old with insulin pump A 55 year-old with cardiac pacemaker A 28 year-old who weighs 310lbs A 19 year-old with pierced ear

SITUATION; A major concern in the public health is the development of resistant strain of TB bacteria. Mila, a public health nurse, will conduct a research on the compliance of TB clients and will look into the reasons why many clients drop out of the short-course chemotherapy. 326. To meet the aim of the study- to identify the factors that affect TB clients” compliance to therapy- the researcher should? A. Determine the purpose of the study B. Acknowledgement the limitations of the study C. Interview clients who have problem in complying

Fourth Year- BSN

D. Identify the research problem and ask the research questions 327. If the researcher would like to establish the relationship between the respondents educational status and compliance to TB treatment, the dependent variable is? A. B. C. D.

TB clients Compliance to treatment Educational status TB treatment

328. There are 300 TB clients enrolled for DOTS in the municipality. What is the best sampling method to use? A. B. C. D.

Simple random Network Purposive Cluster

329. If another researcher would like to do the same study, which of the following is correct? A. He/she should do a meta analysis B. He/she should just barrow the review of literature part of the proposal to save time C. He/she can replicate the study D. It is not ethical to be copying somebody’s study 330. If the department of health (DOH) would like to do a similar study on a national scale, what would be the most appropriate sampling strategy? A. B. C. D.

Probability Simple Cluster Stratified

SITUATION; Mr. Mike Delos Reyes, R.N., is assigned to the medical ward. Severeal clients are admitted for treatment ofvarious conditions. 331. The hallmark sign for the diagnosis of diabetes mellitus (DM) is the presence of? A. B. C. D.

Ketonuria Acetone odor of breath Kussmaul’s breathing Hyperglycemia

Page 33

500 Oncolgy Nursing Questionnaires’ 332. Following stapedectomy with insertion of middle car prosthesis, which of the following instructions should NOT be given to the client? A. B. C. D.

Avoid flying in small planes Avoid deep see diving Do not blow your nose until it is healed You will immediately notice an improvement

333. Which of the following methods as best to obtain the accurate amount of residual urine? A. remove the catheter slowly to ensure drainage of the urine near the exit of the bladder B. Press the lower abdomen while withdrawing the catheter C. Quickly full the catheter so any residual urine is excreted abruptly D. Slowly move the catheter in and out of the bladder 334. Which of the following manifestation is most definitive of pheochromocytoma? A. B. C. D.

Headache Hypertension Hyperglycemia Hypermetabolism

335. Which of the following characteristic of pain indicates gall bladder disease? A. Left epigastric pain with Cullen’s sign B. Right lower quadrant pain with bloody diarrhea C. Right upper quadrant pain that radiates to the shoulder blade D. Lower sterna pain that resembles indigestion SITUATION; Mrs. Maria Delos Angeles, 45 years- old was admitted due ti sudden weakness of the lower extremities. The nurse is assessing the client. 336. How should the nurse assess the trigeminal nerve/ A. Ask the client to say “ah” and assess the swallowing ability B. Used a needle to assess for facial sensation

C.

Assess the client sense of taste in the posterior tongue D. Use a tongue blade to check for the gag reflex 337. Which of the following intervention is the highest priority of the nurse when carrying for a client with myasthenia gravis? A. B. C. D.

Controlling the blood pressure Preventing injury Maintaining an airway Promoting bowel elimination

338. A client with myasthenia gravis was admitted due to muscle weakness and bradycardia. Which drug will the nurse anticipate the doctor to order? A. B. C. D.

Atropine sulfate Neostigmine Pyridostigmine Diazepam ( Valium)

339. Muscle weakness also occurs in client with multiple sclerosis. Following treatment with steroid, an assessment finding in a client with multiple sclerosis that indicates a positive response to therapy is decreased: A. Mood swing B. Muscle rigidity C. Pain in the lower extremities D. Visual problem 340. Mask-like faces occurs in clients with myasthenia gravis nd Parkinson disease. which of the following nursing diagnosis us the highest priority for a client with Parkinson’s disease? A. B. C. D.

Altered nutrition; Less than body requirement Risk for injury Risk for aspiration Altered bowel elimination

SITUATION; In the emergency department, thenurse is obtaining the history of several clients lined up for admission. 341. While playing in the garden, a 10 year-old boy had a foreign object stuck in his left eye. which is intervention is appropriate? A. immediately irrigate the eye with warm water

Fourth Year- BSN

Page 34

500 Oncolgy Nursing Questionnaires’ B. Cover the eye loosely with sterile gauze and refer to the doctor C. Try to remove the object by blowing into the eye D. Wait for the physician before touching the eye 342. Which of the following nursing diagnosis is of the highest priority for a client with Addison’s disease? A. B. C. D.

Fluid and electrolyte imbalance Infection Fluid volume excess Altered cardiac output

343. The client has been receiving one liter of 5% dextrose in normal saline. The total amount of this solution provides how many calories? A. B. C. D.

100 200 350 370

344. Which nursing diagnosis is of highest priority for a client with osteoarthritis? A. B. C. D.

Impaired verbal communication ineffective breathing pattern Risk for injury Impaired nutrition: less than body requirement

345. Which question is most appropriate to ask a client to obtain information about his/her chest pain? A. B. C. D.

“Do you feel some pain” “Do you feel some throbbing” “Tell me about your pain “ “Were you doing something when the pain is started”

346. Cancer occurs when: a. b. c. d.

Normal cells multiple abnormally Normal cells mutate into abnormal cells Abnormal cells become tumor cells Abnormal cells are exposed to toxins

347. During the past 20 years which type of cancer has had an increase in mortality rate?

Fourth Year- BSN

a. b. c. d.

Cevix Stomach Skin melanomas Bladder

348.Cancer death are most likely to occur in which group? a. b. c. d.

Children and adults Young adults Middle adults Older adults

349.Which of the following statements applies to all of the present theories about the etiology of cancer? a. b. c. d.

Cancer occurs only in young people There is impairment of the immune system with cancer In cancer, proteins inhibit the growth of cells In cancer, the actions of oncogenes are repressed

350.Mare Larrabee has cervical cancer. She asks the nurse if it is possible that the cancer was caused by a virus. The correct answer by the nurse is: a. b. c. d.

“Probably not, that is only one many theories.” “All cancers have a viral etiology” “I am not sure- you will have to ask your doctor.” “Some cancers such as yours have a strong association of viruses.”

351.In which of the following cancer is heredity not recognized as a risk factor? a. b. c. d.

Breast cancer Colon cancer Malignant melanoma Brain cancer

352.Which of the following clients probably has the least occupational risk of developing cancer? a. b. c. d.

A coal miner A carpenter An oncology nurse An English teacher

353.Which of the following is characteristic of a benign neoplasm?

Page 35

500 Oncolgy Nursing Questionnaires’ a. b. c. d.

Rapid growth Invasive Noncohesive Well-defined borders

354.Tina Blackey was recently diagnosed with breast cancer. She ask the nurse what steps she can take to boost her immune system. The response by the nurse is: a. b. c. d.

“Refuse chemotherapy; it weakness the immune system” “Learn techniques to relieve or manage stress more effectively” “Ask your doctor to prescribe an antidepressant” “Research herbal remedies”

355.Joan Shipley’s breast cancer has metastasized to lymph nodes. Which statement is correct regarding this condition? a. b. c. d.

Metastasis means that Ms. Shipley will eventually die from cancer The cancer has spread only to the lymph nodes Chemotherapy and radiation will be only palliative in this situation Aggressive treatment may reduce risks of further spread of the cancer

356.Which lab test is it most important to monitor for a client with lung cancer that has ectopic sites? a. b. c. d.

T3 T4 Serum glucose D. blood culture

357.Which statement about cancer is false? a. b. c. d.

Chronic cancer pain maybe treated-related Fear and hopelessness maybe contribute to suffering of the client with cancer Chronic cancer pain may affect objective manifestations Pain relief is not a goal of cancer care

358.The MD has told a client that the results of the biopsy reveal a papilloma. This statement means that the client has:

Fourth Year- BSN

a. b. c. d.

A cancer Metastatic disease A malignancy A benign tumor

359.In the TNM classification system, the N stands for: a. b. c. d.

No metastasis Node involvement Numerical grade Name

360.Which of the following explanations should the nurse utilize regarding the magnetic resonance imaging test used to diagnose a suspected brain tumor? a. b.

c.

d.

“This test uses sound waves” “Before this test, , the physician will inject you with a radiopaque dye and then films will be taken” “The machine for this test is like a tube, and you will lie inside. It may make thumping noises” “You must drink a radioactive substance before this test”

361.A client sys, “The doctor explained so many things to me that I forgot what she said about the endoscopy test I’m having tomorrow.” The nurse should explain that in this test there is a direct visualization of the: a. b. c. d.

Sigmoid colon Upper gastrointestinal tract Urethra Tracheobronchial tree

362.Jean Lawrence is undergoing diagnostic testing as a part of the staging process. She tells the nurse, “I am so worried they will find cancer everywhere: what will happen if I die before my children are grow up?” The most appropriate response by the nurse is: a. b. c. d.

There is no way of knowing what the future holds.” “No matter what happens, your children will be fine.” “You sound like you are afraid.” “Don’t worry until you know the results.”

Page 36

500 Oncolgy Nursing Questionnaires’ 363.Which of the following client statements indicate the need for further client-teaching about chemotherapy? a. b. c. d.

“Chemotherapy is local treatment.” “I may experience nausea, vomiting and hair loss during the treatments.” “I may receive both oral and IV medications.” “I may receive a combination of drugs over different periods of time.”

364.Which of the following are is true about the cell-kill hypothesis? a. b. c. d.

It explains the body’s mechanism of protection fro carcinogens It forms the basis of selecting specific classes of chemotherapeutic drugs It accounts the severity of side effects. It is the reason why several courses of chemotherapy are given.

365.An example of a chemotherapy catheter that is tunneled under the skin on the chest into the subclavian vein is: a. b. c. d.

Hickman Peripherally inserted central catheter (PICC) Mediport Port-a-cath

366.Which of the following nursing interventions is contraindicated for a client receiving Cytoxan (cycloplasphamide) a. b. c. d.

Monitor client’s WBC, BUN, and liver enzyme lab data Teach client about ways to manage hair loss Restrict daily fluid intake to 1L during treatment Encourage the client to use relaxation techniques

367.Assessing for arrythmiasis, gallops, and congestive heart failure is an especially important nursing intervention for a client receiving: a. b. c. d.

Vinblastin (Velban) Doxuribican (Adriamycin) 5-Fluorouracil (5-FU) Vincristine (Onvocin)

Fourth Year- BSN

368.When preparing and administering chemotherapy and disposing of the equipment, the nurse should: a. b. c. d.

Use universal precautions Wear gloves Wear gloves, a mask and a gown Wear gloves and mask

369.Marlene Marrison is scheduled to start chemotherapy. Which statement by Mrs. Marrison indicates the need for further client-teaching? a. b. c. d.

“I will have to drink plenty of fluids each day.” “Visits from my school-aged grandchildren will help my spirits.” “I will need extra rest during this period of time.” “I may need a extra help at home to clean and prepare meals.”

370.Shelley Nordant is scheduled for surgery remove a cancerous tumor. She remarks to the nurse, “Why have surgery? I am going to have a chemotherapy just the same.” The most appropriate response by the nurse is to: a. b. c. d.

Tell Mrs. Nordant that surgery must be performed. Encourage her to confer with the surgeon and the oncologist about the surgery Tell her that the combination of surgery and radiation may lead to cure Document her concerns and give her reassurance

371. Jeovina, with advanced breast cancer is prescribed tamoxifen (Nolvadex). When teaching the client about this drug, the nurse should emphasize the importance of reporting which adverse reaction immediately? a. Vision changes b. Hearing loss c. Headache d. Anorexia 372. A female client with cancer is being evaluated for possible metastasis. Which of the following is one of the most common metastasis sites for cancer cells? a. Liver b. Colon

Page 37

500 Oncolgy Nursing Questionnaires’ c. Reproductive tract d. White blood cells (WBCs) 373. A 34-year-old female client is requesting information about mammograms and breast cancer. She isn’t considered at high risk for breast cancer. What should the nurse tell this client? a. She should have had a baseline mammogram before age 30. b. She should eat a low-fat diet to further decrease her risk of breast cancer. c. She should perform breast self-examination during the first 5 days of each menstrual cycle. d. When she begins having yearly mammograms, breast self-examinations will no longer be necessary. 374. Nurse Brian is developing a plan of care for marrow suppression, the major dose-limiting adverse reaction to floxuridine (FUDR). How long after drug administration does bone marrow suppression become noticeable? a. 24 hours b. 2 to 4 days c. 7 to 14 days d. 21 to 28 days Mr. Danny Ang, an alcoholic, is admitted to the hospital due to cirrhosis of the liver. 375..

376.

Which of the following is the early symptom that he may manifest and would lead to progression into hepatic coma? A. Polyuria C. Nystagmus B. Dysuria D. Behavioral changes What would include the symptoms indicating that three is progression into hepatic coma? 1. Nystagmus 2. Fetid breath 3. Flapping tremor 4. Fruity tremor 5. Fruity odor breath A. 2 and 4 C. 2 and 3 B. 1 and 2 D. 2 and 3

377. The nurse is preparing for a female client for magnetic resonance imaging (MRI) to confirm or rule out a spinal cord lesion. During the MRI scan, which of the following would pose a threat to the client? a. The client lies still. b. The client asks questions. c. The client hears thumping sounds. d. The client wears a watch and wedding band. 378.. Nina, an oncology nurse educator is speaking to a women’s group about breast cancer. Questions and comments from the audience reveal a misunderstanding of some aspects of the disease. Various members of the audience have made all of the following statements. Which one is accurate? a. Mammography is the most reliable method for detecting breast cancer. b. Breast cancer is the leading killer of women of childbearing age. c. Breast cancer requires a mastectomy. d. Men can develop breast cancer. 379.. Nurse Meredith is instructing a premenopausal woman about breast self-examination. The nurse should tell the client to do her self-examination: a. at the end of her menstrual cycle. b. on the same day each month. c. on the 1st day of the menstrual cycle. d. immediately after her menstrual period. 380.. Nurse Kent is teaching a male client to perform monthly testicular self-examinations. Which of the following points would be appropriate to make? a. Testicular cancer is a highly curable type of cancer. b. Testicular cancer is very difficult to diagnose. c. Testicular cancer is the number one cause of cancer deaths in males. d. Testicular cancer is more common in older men. 381. Rhea, has malignant lymphoma. As part of her chemotherapy, the physician prescribes chlorambucil (Leukeran), 10 mg by mouth daily. When caring for the client, the nurse teaches her about adverse reactions to chlorambucil, such as alopecia. How soon after the first administration of chlorambucil might this reaction occur? a. Immediately b. 1 week c. 2 to 3 weeks d. 1 month 382.

Fourth Year- BSN

What is rationale of giving him low-protein diet? A. Decrease formation of ammonia B. Decrease production of bite C. Increase formation of ammonia

Page 38

500 Oncolgy Nursing Questionnaires’ D. 383..

Decrease the desire for alcohol

Because of ineffective breathing pattern, how would you improve his respiratory status: A. Conserve his strength B. Maintain the desired position C. Elevate foot of bed D. Assist him after the paracentesis

384.. A male client is receiving the cell cycle–nonspecific alkylating agent thiotepa (Thioplex), 60 mg weekly for 4 weeks by bladder instillation as part of a chemotherapeutic regimen to treat bladder cancer. The client asks the nurse how the drug works. How does thiotepa exert its therapeutic effects? a. It interferes with deoxyribonucleic acid (DNA) replication only. b. It interferes with ribonucleic acid (RNA) transcription only. c. It interferes with DNA replication and RNA transcription. d. It destroys the cell membrane, causing lysis. 385.. The nurse is instructing the 35 year old client to perform a testicular self-examination. The nurse tells the client: a. To examine the testicles while lying down b. That the best time for the examination is after a shower c. To gently feel the testicle with one finger to feel for a growth d. That testicular self-examination should be done at least every 6 months 386. A female client with cancer is receiving chemotherapy and develops thrombocytopenia. The nurse identifies which intervention as the highest priority in the nursing plan of care? a. Monitoring temperature b. Ambulation three times daily c. Monitoring the platelet count d. Monitoring for pathological fractures 387.

The nurse knows that a dying patient should be encouraged to talk about death and dying. The therapeutic approach is to: A. Confront Mrs. M with the facts about her illness and impending death to open discussion

Fourth Year- BSN

B. C.

D.

Encourage the family members to raise the issue of dying Verify what the physician has told Mrs. M about her diagnosis and prognosis before initiating a discussion Remain available and listen carefully for cues Mr. M is ready to discuss her impending death

388. Mrs. M begins to deteriorate markedly. She requires frequent pain medication and her oldest daughter, Izza age 20, walks out of the room and crying and says to the nurse, “I can not take this anymore.” What would be the most appropriate response of the nurse? A. “I hope your mother didn’t hear you. She needs your support right now.” B. “It is really difficult to see someone you love suffer. Let us go outside and talk about how you are feeling.” C. “You say that you don’t take it anymore. You should also considered what she is feeling.” D. “She will probably die soon. She is getting weaker everyday.” 389.. The nurse performs one of there roles when she listens to Mrs. M and his family: A. Parent surrogate C. Socializing agent B. Teacher D. Counselor

390. Gian, a community health nurse is instructing a group of female clients about breast self-examination. The nurse instructs the client to perform the examination: a. At the onset of menstruation b. Every month during ovulation c. Weekly at the same time of day d. 1 week after menstruation begins 391.. Nurse Cecilia is caring for a client who has undergone a vaginal hysterectomy. The nurse avoids which of the following in the care of this client? a. Elevating the knee gatch on the bed b. Assisting with range-of-motion leg exercises c. Removal of antiembolism stockings twice daily d. Checking placement of pneumatic compression boots

Page 39

500 Oncolgy Nursing Questionnaires’ 392.. Mina, who is suspected of an ovarian tumor is scheduled for a pelvic ultrasound. The nurse provides which preprocedure instruction to the client? a. Eat a light breakfast only b. Maintain an NPO status before the procedure c. Wear comfortable clothing and shoes for the procedure d. Drink six to eight glasses of water without voiding before the test 393.. A male client is diagnosed as having a bowel tumor and several diagnostic tests are prescribed. The nurse understands that which test will confirm the diagnosis of malignancy? a. Biopsy of the tumor b. Abdominal ultrasound c. Magnetic resonance imaging d. Computerized tomography scan 394.Trisha, a 56-years old executive consults her gynecologist due to vaginal bleeding. Diagnostic D & C reveals uterine cancer, Stage II. She is scheduled for total abdominal hysterectomy with bilateral salpingoophorectomy. .

395.

396..

Trisha expresses her fear about the surgery. Which of the following statements is the beast response of the nurse? A. “Let us focus on your post-operative care” B. “Your gynecologist is very competent. You are surely in safe hands” C. “Tell me about your fears. I am here to listen” D. “Do not be afraid because it may affect your recovery” Which of the following is a priority nursing diagnosis immediately after the surgery? A. Disturbance in body image related to loss of uterus B. Fluid volume deficit to surgery C. Abdominal pain related to surgical incision D. Altered breathing pattern related to anesthesia Three days post surgery, the nurse observe that Trisha understands her disease condition when she remarks that Stage II uterine cancer. A. Involves corpus and cervix

Fourth Year- BSN

B. C. D.

Involves bladder, rectum and outside pelvic Extends outside corpus but not outside pelvic Is confined to corpus only

397.. A female client diagnosed with multiple myeloma and the client asks the nurse about the diagnosis. The nurse bases the response on which description of this disorder? a. Altered red blood cell production b. Altered production of lymph nodes c. Malignant exacerbation in the number of leukocytes d. Malignant proliferation of plasma cells within the bone 398.. Nurse Bea is reviewing the laboratory results of a client diagnosed with multiple myeloma. Which of the following would the nurse expect to note specifically in this disorder? a. Increased calcium b. Increased white blood cells c. Decreased blood urea nitrogen level d. Decreased number of plasma cells in the bone marrow 399.. Vanessa, a community health nurse conducts a health promotion program regarding testicular cancer to community members. The nurse determines that further information needs to be provided if a community member states that which of the following is a sign of testicular cancer? a. Alopecia b. Back pain c. Painless testicular swelling d. Heavy sensation in the scrotum 400 The male client is receiving external radiation to the neck for cancer of the larynx. The most likely side effect to be expected is: a. Dyspnea b. Diarrhea c. Sore throat d. Constipation 401. Nurse Joy is caring for a client with an internal radiation implant. When caring for the client, the nurse should observe which of the following principles? a. Limit the time with the client to 1 hour per shift b. Do not allow pregnant women into the client’s room c. Remove the dosimeter badge when entering the client’s room

Page 40

500 Oncolgy Nursing Questionnaires’ d. Individuals younger than 16 years old may be allowed to go in the room as long as they are 6 feet away from the client

402. Which of the following statements best describes therapy? a. Bradytherapy involves placing radioactive materials at some distance from the tumor b. Bradytherapy involves placing radioactive materials directly into tumor site c. Bradytherapy is a holistic, nontraditional cancer treatment d. Bradytherapy is also referred to teletherapy 403. Which of the following nursing actions is most appropriate when caring for client with unsealed implanted radioisotope? a. Wear an abdominal apron b. Have the other client in the room wear monitoring device to measure exposure c. Pick up any dislodged implants found in bed linen with your sterile gloved hand d. Dispose of client’s body fluid in specially marked containers

404. Which of the following statements indicates that the client-teaching prior to an external radiation was successful? a. “If I have pain at my treatment site, I will apply an ice pack.” b. “I will not have any intimate sexual contact while I’m receiving treatment.” c. “I should protect my skin from sun exposure during treatments.” d. “I will wash the skin in the treatment area daily using soap and hot water 405. George Houng is receiving combination therapy of alpha-interferon with another chemotherapeutic agent. Which statement indicates that the client and family require further teaching from the nurse/

Fourth Year- BSN

a. “I will observed my husband for any chances of mental status.” b. “I understand how to give the drug subcutaneously.” c. “I can have serve flulike symptoms while taking these drugs.” d. “I will limit my fluid intake.” 406. A nurse using the Karnofsky scale to assess a client with cancer is evaluating the client: a. Amount of pain b. Stress level c. Performance status d. Nutritional intake 407. Which finding would support a diagnosis of poor nutrition/ a. Oral mucous membranes pink-red and moist b. Tongue bright to dark red and swollen c. Slight hair loss d. Acne 408. Stuart Eggleton, a client with prostate cancer, is trembling and is pacing his room. His blood pressure and pulse are slightly elevated. Mr. Eggleton may be experiencing: a. Some type of drug reaction b. Anxiety c. Body image disturbance d. Fear of the unknown 409. Which of the following statements indicates that the client has misunderstood client-teaching about alopecia from chemotherapy? a. “I can wear a colorful scarf of turban.” b. “I have already bought a wig.” c. “My neighbor will be driving me to the ‘Look Good, Feel Better’ program.” d. “After the chemotherapy is finished, my hair will grow back as the same it was before the treatment.” 410. Which finding indicates a possible infection in the client with cancer who is immunosuppressed? a. A decreased respiratory rate b. A decreased pulse rate Page 41

500 Oncolgy Nursing Questionnaires’ c. A normal temperature d. An increased blood pressure 411. The partner of a client with cancer has been provided with guidelines regarding when to call for help. Which of the following statements by the client’s partner indicates further teaching about the guidelines is needed? a. “I’ll contact the physician if I see ant new bleeding from her rectum.” b. “If her oral temperature is 100F, I’ll notify the physician.” c. “If she becomes withdrawn and cries frequently, I’ll notify the physician.” d. “I’ll contact the physician If I notice a change in her eating patterns.” 412. Which f the following nursing assessment would indicate an improvement in a cancer client with nursing diagnosis Altered Nutrition: Less than the body Requirements? a. Serum albumin less than 3.0 b. A PPD response of more than 5 mm of induration c. A decrease in urinary creatinine level d. An anthropometric measurement of 75% of the standard 413. Which of the following nursing intervention would be most helpful in managing nausea and vomiting of a client with cancer? a. Instruct the client to drink plenty of fluids each meal b. Instruct the client to lie down for an hour of eating c. Encourage the client to eat dry foods d. Instruct the client to follow high-fat food plan 414. Which of the following nursing actions is contraindicated for the client with leukemia? a. Assess the client’s oral mucous membranes daily b. Instruct the client to brush and floss the teeth three ties per day Fourth Year- BSN

c. Prevent dry mouth by using moisturizing agents such as Blistex d. Teach client to use smooth rather than sharp utensils for eating. 415. Irina Landrigan is 49 year-old client who has just undergone a mastectomy. When the nurse is performing the first dressing change, Mrs. Landrigan states, “I just do not want to look at it.” The most appropriate response by the nurse is: a. “You are going to have to look at it some time.” b. “You do not have to look at your incision.” c. “You will eventually be able to look at it.” d. “Every woman with this kind of surgery feels that way.” 416. The doctor has told Javier Rodriguez and his family that has about 6 to 9 months to live. Yhe family asks the nurse about making funeral arrangements ahead of time. The most appropriate response by the nurse is: a. “Many people do make arrangements ahead, but you might upset Mr. Rodriguez if you tell him about the plans.” b. “If you would like to do this, you should try to include Mr. Rodriguez in the planning.” c. “Don’t give up hope; he may still respond to therapy.” d. “You should wait until Mr. Rodriguez dies before making funeral plans.” 417. Which strategy would not be recommended to a cancer client with problems eating? a. Eat icy-cold foods b. Eat frequent, small meals c. Eat semisoft and liquid foods d. Eat spicy, warm foods 418. Which of the following statements indicates that the teaching about the American Cancer Society’ s dietary guidelines to prevent was understood by the team? a. “I should take vitamin supplements ever day.” Page 42

500 Oncolgy Nursing Questionnaires’ b. “My food pattern should include serving of salt-cured foods four times a week.” c. “I will include high-fat foods in my meal plan.” d. “eating more high-fiber foods will be better for me.” 419. Dana Biencarz has a lung cancer and is experiencing facial and arm edema. The nurse should suspected that Mrs. Biancarz has: a. Allergic reaction b. Pericardial effusion c. Sepsis d. Superior vena cava syndrome 420. Rita Lopez has metastatic breast cancer and is admitted to the hospital for pain management. During the night she becomes incontinent, which has never occurred before. The appropriate action by the nurse is to: a. Recognize that the incontinence occurred as a result of oversedation b. Notify the physician that the spinal cord compression is suspected c. Realize that incontinence can occur in metastatic bone disease d. Wait until the morning to see if the incontinence recurs

421.. A cervical radiation implant is placed in the client for treatment of cervical cancer. The nurse initiates what most appropriate activity order for this client? a. Bed rest b. Out of bed ad lib c. Out of bed in a chair only d. Ambulation to the bathroom only 422.. A female client is hospitalized for insertion of an internal cervical radiation implant. While giving care, the nurse finds the radiation implant in the bed. The initial action by the nurse is to: a. Call the physician b. Reinsert the implant into the vagina immediately c. Pick up the implant with gloved hands and flush it

Fourth Year- BSN

down the toilet d. Pick up the implant with long-handled forceps and place it in a lead container. 423. The nurse is caring for a female client experiencing neutropenia as a result of chemotherapy and develops a plan of care for the client. The nurse plans to: a. Restrict all visitors b. Restrict fluid intake c. Teach the client and family about the need for hand hygiene d. Insert an indwelling urinary catheter to prevent skin breakdown 424. The home health care nurse is caring for a male client with cancer and the client is complaining of acute pain. The appropriate nursing assessment of the client’s pain would include which of the following? a. The client’s pain rating b. Nonverbal cues from the client c. The nurse’s impression of the client’s pain d. Pain relief after appropriate nursing intervention 425.. The nurse is interviewing a male client about his past medical history. Which preexisting condition may lead the nurse to suspect that a client has colorectal cancer? a. Duodenal ulcers b. Hemorrhoids c. Weight gain d. Polyps 426.. Nurse Amy is speaking to a group of women about early detection of breast cancer. The average age of the women in the group is 47. Following the American Cancer Society guidelines, the nurse should recommend that the women: a. perform breast self-examination annually. b. have a mammogram annually. c. have a hormonal receptor assay annually. d. have a physician conduct a clinical examination every 2 years. 427. A male client with a nagging cough makes an appointment to see the physician after reading that this symptom is one of the seven warning signs of cancer. What is another warning sign of cancer? a. Persistent nausea b. Rash c. Indigestion d. Chronic ache or pain 428.. For a female client newly diagnosed with radiationinduced thrombocytopenia, the nurse should include

Page 43

500 Oncolgy Nursing Questionnaires’ which intervention in the plan of care? a. Administering aspirin if the temperature exceeds 102° F (38.8° C) b. Inspecting the skin for petechiae once every shift c. Providing for frequent rest periods d. Placing the client in strict isolation 429.. Nurse Lucia is providing breast cancer education at a community facility. The American Cancer Society recommends that women get mammograms: a. yearly after age 40. b. after the birth of the first child and every 2 years thereafter. c. after the first menstrual period and annually thereafter. d. every 3 years between ages 20 and 40 and annually thereafter. 430. A male client is in isolation after receiving an internal radioactive implant to treat cancer. Two hours later, the nurse discovers the implant in the bed linens. What should the nurse do first? a. Stand as far away from the implant as possible and call for help. b. Pick up the implant with long-handled forceps and place it in a lead-lined container. c. Leave the room and notify the radiation therapy department immediately. d. Put the implant back in place, using forceps and a shield for self-protection, and call for help.

431. The nurse is admitting a client with laryngeal cancer to the Nursing unit. The nurse assesses for which most common risk factor for this type of cancer? a. Alcohol abuse b. Cigarette smoking c. Use of chewing tobacco d. Exposure to air pollutants 432. The nurse recognizes which of the following statements as accurately reflecting a risk factor for breast cancer? a. Mother affected by cancer before 60 years of age b. Onset of menses before 14 years of age c. Multiparity d. No alcohol consumption 433. Ductal lavage is used for

Fourth Year- BSN

a. women at higher risk for benign proliferative breast disease. b. women at low risk for breast cancer. c. screening women over age 65. d. women with breast implants.

434. The nurse recognizes which of the following statements as accurately reflecting a risk factor for breast cancer? a. Mother affected by cancer before 60 years of age b. Onset of menses before 14 years of age c. Multiparity d. No alcohol consumption 435. The nurse is teaching a client about the risk factors associated with colorectal cancer. The nurse determines that further teaching related to colorectal cancer is necessary if the client identifies which of the following as an associated risk factor? a. Age younger than 50 years b. History of colorectal polyps c. Family history of colorectal cancer d. Chronic inflammatory bowel disease 435. Answer: A Rationale: Colorectal cancer risk factors include age older than 50 years, a family history of the disease, colorectal polyps, and chronic inflammatory bowel disease. 436. When teaching a client about the signs of colorectal cancer, Nurse Trish stresses that the most common complaint of persons with colorectal cancer is: a. Abdominal Pain b. Hemorrhoids c. Change in caliber of stools d. Change in bowel habits 436. Answer: D Rationale: Constipation, diarrhea, and/or constipation alternating with diarrhea are the most common symptoms of colorectal cancer

438. The nurse is teaching a client about the risk factors associated with colorectal cancer. The nurse determines that further teaching related to colorectal cancer is

Page 44

500 Oncolgy Nursing Questionnaires’ necessary if the client identifies which of the following as an associated risk factor? a. Age younger than 50 years b. History of colorectal polyps c. Family history of colorectal cancer d. Chronic inflammatory bowel disease 438. Answer: A Rationale: Colorectal cancer risk factors include age older than 50 years, a family history of the disease, colorectal polyps, and chronic inflammatory bowel disease.

439. The 85-year-old male client diagnosed with cancer of the colon asks the nurse, “Why did I get this cancer?” Which statement is the nurse’s best response? a. Cancer of the colon is associated with a lack of fiber in the diet. b. Cancer of the colon has a greater incidence among those younger than age 50 years. c. Cancer of the colon has no known risk factors. d. Cancer of the colon is rare among male clients.

441.The nurse is caring for a client diagnosed with squamous cell skin cancer and writes a psychosocial problem of “fear.” Which Nursing interventions should be included in the plan of care? a. Explain to the client that the fears are unfounded. b. Encourage the client to verbalize the feeling of being afraid. c. Have the HCP discuss the client’s fear with the client. d. Instruct the client regarding all planned procedures. 441.Answer: B Rationale: This is the most commonly written therapeutic communication goal. This addresses the client’s concerns.

442. The 33-year-old client diagnosed with Stage IV Hodgkin’s lymphoma is at the five (5)- year remission mark. Which information should the nurse teach the client?

439. Answer: A Rationale: A long history of low-fiber, high-fat, high protein diets results in a prolonged transit time. This allows the carcinogenic agents in the waste products to have a greater exposure to the lumen of the colon.

a. Instruct the client to continue scheduled screenings for cancer. b. Discuss the need for follow-up appointments every five (5) years. c. Teach the client that the cancer risk is now the same as for the general population. d. Have the client talk with the family about funeral arrangements.

440. The nurse has been giving instructions to Mrs. S., a c5-year-old white female, about preventing skin cancer. Which statement best indicates Mrs. S.’s understanding of skin cancer risk factors?

442. Answer: A Rationale: The five (5)-year mark is a time for celebration for clients diagnosed with cancer, but the therapies can cause secondary malignancies and there may be a genetic predisposition for the client to develop cancer. The client should continue to be tested regularly.

a. “My father was treated for melanoma, but my mom says not to worry.” b. “I really need to use sunscreen–even in winter.” c. “I guess because I am dark complected I will be more prone to developing skin cancer.” d. “I used to lay in the sun all the time–now I just go to the tanning bed.” 440. Correct Answer: B Rationale: Almost all cases of basal and squamous cell skin cancer diagnosed each year in the United States are considered to be sun-related.

Fourth Year- BSN

443.Which clinical manifestation of Stage I nonHodgkin’s lymphoma would the nurse expect to find when assessing the client? a. Enlarged lymph tissue anywhere in the body. b. Tender left upper quadrant. c. No symptom in this stage. d. Elevated B cell lymphocytes on the CBC.

Page 45

500 Oncolgy Nursing Questionnaires’ 444. The nurse is admitting a client with rule-out Hodgkin’s lymphoma. When the nurse assesses the client, which data would support this diagnosis? a. Night sweats and fever without “chills.” b. Edematous lymph nodes in the groin. c. Malaise and complaints of an upset stomach. d. Pain in the neck area after a fatty meal.

445. A client admitted with newly diagnosed with Hodgkin’s disease. Which of the following would the nurse expect the client to report? a. Lymph node Pain b. Weight gain c. Night sweats d. Headache 446. he client is diagnosed with Cancer of the head of the Pancreas. When assessing the patient, which signs and symptoms would the nurse expect to find? a. Clay-colored stools and dark urine. b. Night sweats and fever. c. Left lower abdominal cramps and tenesmus. d. Nausea and coffee-ground emesis. 447. The nurse is planning a program for clients at a health fair regarding the prevention and early Detection of Cancer of the Pancreas. Which self-care activity should the nurse teach that is an example of primary Nursing care? a. Monitor for elevated blood glucose at random intervals. b. Inspect the skin and sclera of the eyes for a yellow tint. c. Limit meat in the diet and eat a diet that is low in fats. d. Instruct the client with hyperglycemia about insulin injections. 448. The community nurse is conducting a health promotion program and the topic of the discussion relates to the risk factors for gastric cancer. Which risk factor, if identified by a client, indicates a need for further discussion? a. Smoking b. A high-fat diet c. Foods containing nitrates d. A diet of smoked, highly salted, and spiced food

Fourth Year- BSN

449. The occupational health nurse is preparing a presentation to a group of factory workers about preventing colon cancer. Which information should be included in the presentation? a. Wear a high filtration mask when around chemicals. b. Eat several servings of cruciferous vegetables daily. c. Take a multiple vitamin every day. d. Do not engage in high-risk sexual behaviors. 450. A client is being evaluated for cancer of the colon. In preparing the client for barium enema, the nurse should: a. Give laxative the night before and a cleansing enema in the morning before the test b. Render an oil retention enema and give laxative the night before c. Instruct the client to swallow 6 radiopaque tablets the evening before the study d. Place the client on CBR a day before the study 451.The nurse is caring for the client scheduled for an abdominal perineal resection for Stage IV colon cancer. When preparing the plan of care during surgery, which client problem should the nurse include in the plan? a. Fluid volume deficit. b. Impaired tissue perfusion. c. Infection of surgical site. d. Immunosuppression.

452. The nurse is reviewing the preoperative orders of a client with a colon tumor who is scheduled for abdominal perineal resection and notes that the physician has prescribed neomycin (Mycifradin) for the client. The nurse determines that this medication has been prescribed primarily: a. To prevent an immune dysfunction b. Because the client has an Infection c. To decrease the bacteria in the bowel d. Because the client is allergic to penicillin The nurse is admitting a male client to a medical floor with a diagnosis of adenocarcinoma of the rectosigmoid colon. Which assessment data support this diagnosis? a. The client reports up to 20 bloody stools per day. b. The client states that he has a feeling of fullness after a heavy meal. c. The client has diarrhea alternating with constipation.

Page 46

500 Oncolgy Nursing Questionnaires’ d. The client complains of right lower quadrant Pain with rebound tenderness. 454. During the admission assessment of a client with advanced ovarian cancer, the nurse recognizes which symptom as typical of the disease? a. Diarrhea b. Hypermenorrhea c. Abnormal bleeding d. Abdominal distention 455. The client with ovarian cancer is being treated with vincristine (Oncovin, Vincasar PFS). The nurse monitors the client, knowing that which of the following indicates a side effect specific to this medication? a. Diarrhea b. Hair loss c. Chest Pain d. Numbness and tingling in the fingers and toes

a. Sleep with the head of the bed elevated to prevent increased intracranial pressure. b. Take an analgesic medication for Pain only when the Pain becomes severe. c. Explain that radiation therapy to the head may result in permanent hair loss. d. Discuss end-of-life decisions prior to cognitive deterioration.

459. A client with leukemia is undergoing radiation therapy to the brain and spinal cord. In planning care for this client, the nurse would include which Nursing intervention? a. A scalp ointment to prevent dryness b. Avoiding washing off the target’s marks c. Not allowing the client to use a hat or scarf d. A dandruff shampoo twice daily Marie with acute lymphocytic leukemia suffers from nausea and headache. These clinical manifestations may indicate all of the following except

456. The client with small cell lung cancer is being treated with etoposide (VePesid). The nurse monitors the client during administration, knowing that which of the following indicates a side effect specific to this medication? a. Alopecia b. Chest Pain c. Pulmonary fibrosis d. Orthostatic hypotension

457. A female client with carcinoma of the breast is admitted to the hospital for treatment with intravenously administered doxorubicin (Adriamycin). The client tells the nurse that she has been told by herfriends that she is going to lose all her hair. The appropriate Nursing response is which of the following? a. “Your friends are correct.” b. “You will not lose your hair.” c. “Hair loss may occur, but it will grow back just as it is now.” d. “Hair loss may occur, and it will grow back, but it may have a different color or texture.”

a. effects of radiation b. chemotherapy side effects c. meningeal irritation d. gastric distension 461. The nurse has reviewed a discharge teaching checklist for a client, a 65-year-old male with chronic lymphocytic leukemia (CLL). Which of the following statements by the client would indicate to the nurse that further review is necessary? a. “I’m retired, so I can sleep whenever I want.” b. “I’ve got season tickets for all the basketball games.” c. “I’ll call the doctor if I have fever greater than 99°F.” d. “I’m going to teach my grandson how to fish.” 462. The client is diagnosed with chronic lymphocytic leukemia (CLL) after routine laboratory tests during a yearly physical. Which is the scientific rationale for the random nature of discovering the illness? a. CCL is not serious, and clients die from other causes first. b. There are no symptoms with this form of leukemia. c. This is a childhood illness and is self-limiting. d. In early stages of CLL the client may be asymptomatic.

458. The client diagnosed with leukemia has central nervous system involvement. Which instructions should the nurse teach?

Fourth Year- BSN

Page 47

500 Oncolgy Nursing Questionnaires’ 463. The client diagnosed with leukemia is being admitted for an induction course of chemotherapy. Which laboratory values indicate a diagnosis of leukemia? a. A left shift in the white blood cell count differential. b. A large number of WBCs that decreases after the administration of antibiotics. c. An abnormally low hemoglobin (Hgb) and hematocrit (Hct) level. d. Red blood cells that are larger than normal. 464. The nurse is assessing a client diagnosed with acute myeloid leukemia. Which assessment data support this diagnosis? a. Fever and infections. b. Nausea and vomiting. c. Excessive energy and high platelet counts. d. Cervical lymph node enlargement and positive acid-fast bacillus.

about chemotherapy is true? A. it is a local treatment affecting only tumor cells B. it affects both normal and tumor cells C. it has been proven as a complete cure for cancer D. it is often used as a palliative measure.

469. . Which is an incorrect statement pertaining to the following procedures for cancer diagnostics? A. Biopsy is the removal of suspicious tissue and the only definitive method to diagnose cancer B. Ultrasonography detects tissue density changes difficult to observe by X-ray via sound waves. C. CT scanning uses magnetic fields and radio frequencies to provide cross-sectional view of tumor D. Endoscopy provides direct view of a body cavity to detect abnormality. 470.. A post-operative complication of mastectomy is lymphedema. This can be prevented by

465. Which of the following observations reported by a patient with acute myelogenous leukemia (AML) would the nurse first assess? a. Weakness and fatigue b. Bruising on the arm c. Drainage from a small finger cut d. Mild abdominal Pain 466.. Mr. Perez is in continuous pain from cancer that has metastasized to the bone. Pain medication provides little relief and he refuses to move. The nurse should plan to: A. Reassure him that the nurses will not hurt him B. Let him perform his own activities of daily living C. Handle him gently when assisting with required care D. Complete A.M. care quickly as possible when necessary 467. . A client, who is suspected of having Pheochromocytoma, complains of sweating, palpitation and headache. Which assessment is essential for the nurse to make first? A. Pupil reaction B. Hand grips C. Blood pressure D. Blood glucose 468. . A chemotherapeutic agent 5FU is ordered as an adjunct measure to surgery. Which of the ff. statements

Fourth Year- BSN

A. ensuring patency of wound drainage tube B. placing the arm on the affected side in a dependent position C. restricting movement of the affected arm D. frequently elevating the arm of the affected side above the level of the heart. .471. Which statement by the client indicates to the nurse that the patient understands precautions necessary during internal radiation therapy for cancer of the cervix? A. “I should get out of bed and walk around in my room.” B. “My 7 year old twins should not come to visit me while I’m receiving treatment.” C. “I will try not to cough, because the force might make me expel the application.” D. “I know that my primary nurse has to wear one of those badges like the people in the x-ray department, but they are not necessary for anyone else who comes in here.” \472.. The laboratory results of the client with leukemia indicate bone marrow depression. The nurse should encourage the client to: A. Increase his activity level and ambulate frequently B. Sleep with the head of his bed slightly elevated C. Drink citrus juices frequently for nourishment

Page 48

500 Oncolgy Nursing Questionnaires’ D. Use a soft toothbrush and electric razor

473. Dennis receives a blood transfusion and develops flank pain, chills, fever and hematuria. The nurse recognizes that Dennis is probably experiencing: A. An anaphylactic transfusion reaction B. An allergic transfusion reaction C. A hemolytic transfusion reaction D. A pyrogenic transfusion reaction

474.. A client jokes about his leukemia even though he is becoming sicker and weaker. The nurse’s most therapeutic response would be:

A. Changing the abdominal dressing B. Maintaining patency of the cystotomy tube C. Maintaining patency of a three-way Foley catheter for cystoclysis D. Observing for hemorrhage and wound infection

478. In the early postoperative period following a transurethral surgery, the most common complication the nurse should observe for is: A. Sepsis B. Hemorrhage C. Leakage around the catheter D. Urinary retention with overflow

A. “Your laugher is a cover for your fear.” B. “He who laughs on the outside, cries on the inside.” C. “Why are you always laughing?” D. “Does it help you to joke about your illness?”

479.. Following prostate surgery, the retention catheter is secured to the client’s leg causing slight traction of the inflatable balloon against the prostatic fossa. This is done to:

475. n dealing with a dying client who is in the denial stage of grief, the best nursing approach is to:

A. Limit discomfort B. Provide hemostasis C. Reduce bladder spasms D. Promote urinary drainage

A. Agree with and encourage the client’s denial B. Reassure the client that everything will be okay C. Allow the denial but be available to discuss death D. Leave the client alone to discuss the loss 476.. Rene, age 62, is scheduled for a TURP after being diagnosed with a Benign Prostatic Hyperplasia (BPH). As part of the preoperative teaching, the nurse should tell the client that after surgery: A. Urinary control may be permanently lost to some degree B. Urinary drainage will be dependent on a urethral catheter for 24 hours C. Frequency and burning on urination will last while the cystotomy tube is in place D. His ability to perform sexually will be permanently impaired 477. The transurethral resection of the prostate is performed on a client with BPH. Following surgery, nursing care should include:

Fourth Year- BSN

480.Twenty-four hours after TURP surgery, the client tells the nurse he has lower abdominal discomfort. The nurse notes that the catheter drainage has stopped. The nurse’s initial action should be to: A. Irrigate the catheter with saline B. Milk the catheter tubing C. Remove the catheter D. Notify the physician 481. The nurse would know that a post-TURP client understood his discharge teaching when he says “I should:” A. Get out of bed into a chair for several hours daily B. Call the physician if my urinary stream decreases C. Attempt to void every 3 hours when I’m awake D. Avoid vigorous exercise for 6 months after surgery

Page 49

500 Oncolgy Nursing Questionnaires’ 482. A patient with relapsing Hodgkin’s disease presents with weight gain, foot ulcers, vision problems, elevated blood sugar, oral candidiasis, and new onset of wildly swinging mood changes. What is the most likely etiology of this patient’s psychiatric symptoms? (A) Adverse effects of bleomycin (B) Adverse effects of prednisone (C) Adverse effects of vincristine (D) Normal psychiatric response to having cancer 483. A 41-year-old pregnant woman sees her obstetrician because of new-onset vaginal bleeding. Although she is only 4 months pregnant, her doctor notes that her uterus is the size usually seen at 6 months of gestation. Maternal blood works shows a -human chorionic gonadotropin (hCG) level >5 times the upper limit of normal. If left untreated, what is a possible consequence of the patient’s condition? (A) Choriocarcinoma (B) Coma (C) Fetal neural tube defects (D) Ovarian cancer 484. Oncology Nurse Test Questions about Hydrops fetalis that occurs in the setting of a certain type of thalassemia. What is the underlying mechanism leading to this event? (A) Excess -globin chains binding tighter to oxygen (B) Excess -globin chains binding weaker to oxygen (C) Excess -globin chains binding tighter to oxygen (D) Excess gamma-globin chains binding tighter to oxygen 485. A 56-year-old man who is a health care worker presents to his physician with vague abdominal discomfort. A physical examination reveals a tender liver, palpable to 6 cm below the costal margin and scleral icterus. His laboratory studies are significant for an aspartate aminotransferase activity of 200 U/L and an alanine aminotransferase activity of 450 U/L. A CT scan of the abdomen shows a dominant solid nodule in the liver. The marker most likely to be elevated in this patient is also a good indicator of which of the following malignancies? (A) Choriocarcinoma (B) Colorectal carcinoma (C) Melanoma

Fourth Year- BSN

(D) Yolk sac carcinoma 486. A 57-year-old man presents to his physician with a 4month history of worsening fatigue and generalized weakness. Further questioning reveals that his clothes fit him more loosely now than they had in the past. Physical examination reveals generalized lymphadenopathy and hepatosplenomegaly. Lymph node biopsy specimens are sent to the pathologist with the presumptive diagnosis of lymphoma. Which of the following types of neoplastic cell is most common in non-Hodgkin’s lymphoma? (A) B lymphocyte (B) Myeloblast (C) Plasma cell (D) Reed-Sternberg cell

487.. A 29-year-old man presents to his primary care physician with a painless testicular mass. Laboratory studies show an elevated serum human chorionic gonadotropin level. Which of the following is the most likely site of nodal metastasis in this tumor? (A) Deep inguinal lymph nodes (B) External iliac lymph nodes (C) Gluteal lymph nodes (D) Para-aortic lymph nodes 488. A 28-year-old woman comes to the physician concerned about an excessive amount of bleeding from her gums when she brushes her teeth. Her laboratory results show an increased partial thromboplastin time and an increased bleeding time, but are otherwise unremarkable. Which of the following treatments will most likely alleviate this patient’s symptoms? (A) Cryoprecipitate (B) Factor VIII concentrate (C) Fresh frozen plasma( correction : vitamin K) (D) Low-molecular-weight heparin 489. . Nurse Questions about a 29-year-old woman, who is 32 weeks’ pregnant and has been in the hospital for 3 days because of pyelonephritis, starts oozing blood from her intravenous lines and bleeding from her gums. Petechiae are also noted in her skin. Laboratory tests show a platelet count of 98,000/mm3, hematocrit of 38%, WBC count of 8000/mm3, and a prolonged prothrombin time. What other laboratory anomaly would also be expected? (A) Elevated D-dimer levels

Page 50

500 Oncolgy Nursing Questionnaires’ (B) Elevated factor VII levels (C) Elevated fi brinogen levels (D) Elevated protein C levels 490. A 62-year-old woman presents to the clinic complaining of frequent bleeding while brushing her teeth and easy bruising. She reports she recently had pneumonia and was treated with a broad-spectrum antibiotic. Laboratory tests show: Prothrombin time: 18 seconds Partial thromboplastin time: 37 seconds Platelet count: 231,000/mm3 Hematocrit: 37% WBC count: 4800/mm3 The cofactor that is deficient in this patient is needed for the carboxylation of glutamate residues of which of the following? (A) Factors II, VII, VIII, and X (B) Factors VII, VIII, IX, and XII (C) Proteins C and S and factors IX, X, XI, and XII (D) Proteins C and S, prothrombin, and factors VII, IX, and X 491. Risk factors associated with colorectal cancer include which one of the following? a. Irritable bowel syndrome. b. Low intake of dietary fiber. c. Low intake of red meat. d. Chronic aspirin therapy. 492. Colorectal cancer: a. Is more common in males. b. Is the most common cancer in the UK. c. Most commonly occurs in cecum. d. Commonly arises from pre-existing adenomas. 493. Characteristics presenting features of left sided colorectal tumors include: a. Acute large bowel obstruction. b. Iron deficiency anemia. c. Painless abdominal mass. d. Foul smelling stools which are difficult to flush. 494. Features associated with poor prognosis in colorectal cancer include: a. Adenocarcinoma cell type. b. Rectal bleeding at presentation. c. Presence of involved lymph nodes. d. Lymphocytic response of tumor. 495. . Surgery for liver metastases from colorectal cancer is:

Fourth Year- BSN

a. b. c. d.

Associated with a 10% mortality. Contraindicated if 1 lesion is present. Associated with 16-40% 5year survival. Usually followed with consolidation radiotherapy. 496. 2.Fluorouracil based chemotherapy: a. Is indicated for all patients with Duke’s B Tumours. b. In combination with capecitabine improves survival to around 18months in metastatic disease. c. I s likely t impair quality of life when used in the metastatic setting. d. Improves 5years survival rates for patients with Duke’s C Tumours by approximately 7%. 497. . Which of the following is a feature of colorectal cancer? a. In Duke’S 90% patients are cured by surgery alone. b. Distant metastases are present in less than 5% of patients in presentation. c. Chemotherapy has no effect on survival. d. In Dukes’ C disease
View more...

Comments

Copyright ©2017 KUPDF Inc.
SUPPORT KUPDF